Page - 1: Prefer Calling Sir Obiero Amos at 0706 851 439 For F1-F4 All Subjects Notes

You might also like

Download as pdf or txt
Download as pdf or txt
You are on page 1of 172

F1 NEW GEOGRAPHY NOTES

Page | 1
Prefer Calling Sir Obiero Amos @ 0706 851 439 for F1-F4 All Subjects Notes
INTRODUCTION TO GEOGRAPHY
The meaning of geography.
The word ‘Geography’ has its origin in the Greek language; it is a two word
combination of Geo which means earth and Graphein or grapho which means
to draw, describe or to write.
Geopgraphia therefore means to draw, describe or write about the earth.
Geography is the scientific study of the earth as a home of humankind.
It is also the study of the distribution and interrelationship of natural and human
phenomena on the earth’s surface.
Geography is a science that studies how man interacts with the
environment.

Environment
The environment refers all the external conditions which have influence over
the behaviour of an organism (plants, animals and human beings).
There are two divisions of the environment;
a) The physical environment – the natural physical conditions of weather, climate,
vegetation, animals, soils, landforms and drainage.
b) The human environment; including human activities such as farming, forestry,
mining, tourism, settlement, transportation, trade and industry.

Why do we study Geography?


a) "To gain knowledge about our environment and how to control it for
both the present and the future generations."
b) "To understand and explain how man interacts with his surroundings."

c) "It helps us to be aware of the physical features within our environment,


how they are formed, the benefits we get from them and the threats they
pose."
d) It is a career subject. "Geography provides useful skills for becoming a
teacher, surveyor, planner, geologist and environmentalists, so as to earn
a living."
e) “Geography teaches us the basic principles and geographical methods of
studying and solving problems of national development.”
f) “Geography also helps us to acquire positive attitudes and values which
enable us to become useful members of the society. During fieldwork, an
individual is able to develop respect for work especially group work.”
g) “Studying geography of other regions in the world creates international
awareness which facilitates good relations among people.”

THE MAJOR BRANCHES OF GEOGRAPHY


1) Physical geography deals with the study of the natural physical
environment (earth's landscape and atmosphere).
It includes the study of:
a. The earth in relation to the solar system.

b. The shape of the landscape (called Geomorphology)

c. Weather (called Meteorology)

d. Climate (called Climatology)

e. The location of plants and animals (called Biogeography)

f. Rocks (called Geology)

Human and economic geography deals with man's economic activities


such as agriculture, commerce, tourism, industrialization, transport,
commerce.
It has the following sub-branches;
a. Historical geography (How land use has changed over time)

b. Cultural geography (How land is used in different cultures)

c. Demography (the effect of changes in population)

d. Economic geography (How man creates wealth from the


environment)
e. Social geography (How changes in society affect land use)

f. Political geography (How politics affects land use)

g. Settlement geography (How changes in our villages and towns affect


the landscape)

Page | 3
Prefer Calling Sir Obiero Amos @ 0706 851 439 for F1-F4 All Subjects Notes
2) Practical geography is a smaller branch that equips learners with the
practical skills that enhance their understanding and interpretation of
physical and human geographical information.

It includes;
a. Statistical methods

b. Maps and map work.

c. Fieldwork.

d. Photograph work

RELATIONSHIP BETWEEN GEOGRAPHY AND OTHER DISCIPLINES


This relationship comes about when geography applies certain principles and
facts from other subjects to explain or solve certain geographical problems or
when these subjects apply geographical information to explain various
concepts.
1. Geography and Mathematics

• Mathematical principles and formulae are used in Geography to


calculate distances, areas and population densities. E.g. use of graphs
and pie charts.
• Geographical knowledge about bearing and direction is in turn used by
mathematicians to calculate distances around the globe.

2. Geography and history


• Historical geography exists as a branch that ways in which historical
events are affecting human economic activities. Geography attempts to
map these events.
• History uses geographical tools like maps and charts to show
movements of people.

3. Geography and biology


While biology studies organisms, focusing on their anatomy, physiology
and behaviour, geography is interested in their distribution, factors

Page | 4
Prefer Calling Sir Obiero Amos @ 0706 851 439 for F1-F4 All Subjects Notes
influencing the distribution and the influence of the biological
organisms on human activities.

4. Geography and physics


• Physics studies matter, energy, heat, light, gravity and magnetism.
• Geography focuses on heat from the sun as it is responsible for
movement of air, evaporation of water and distribution of moisture in
the atmosphere.
• The earth’s magnetic field, gravity and its vibrations, areas dealt with
by geophysicists, help geographers to understand the causes and effects
of earth quakes.

5. Geography and chemistry.


• Chemistry studies substances, their composition and
behaviour.
• Geography applies chemistry in studying the chemical
composition of, and changes in rocks and soils.

6. Geography and agriculture.


• Agriculture involves cultivation of crops and rearing of
animals. Geography is occupied with studying farming
systems, their distribution and factors affecting farming
activities.

7. Geography and meteorology


• Meteorology deals with atmospheric conditions of a pace and
weather forecasting.
• The meteorological information is important to a geographer
in classification and mapping of climates. Climatology is the
sub branch of geography that deals with this.

Page | 5
Prefer Calling Sir Obiero Amos @ 0706 851 439 for F1-F4 All Subjects Notes
8. Geography and medicine
• While medicine is concerned with disease diagnosis,
prevention and cure, geography attempts to establish factors
influencing spread of disease and the diseases affect economic
activities.

9. Geography and economics


• Economics deals with the study of commodities, focusing on money
and trade.
• Geography similarly focuses on the exploitation of resources, methods
and factors of production, movement of commodities and their
consumption.
A MODEL SHOWING RELATIONSHIP BETWEEN GEOGRAPHY AND OTHER DISCILP LINES

economics.

Human and social sciences

Geography.

Physical and biological sciences

Biology.

Task 1: with the help of your teacher, find some pictures and photographs that
illustrate the different branches of Geography and create a wall display. Also
illustrate the centrality of Geography..

Page | 6
Prefer Calling Sir Obiero Amos @ 0706 851 439 for F1-F4 All Subjects Notes
THE SOLAR SYSTEM AND EARTH
Introduction
The sun, moon and stars are commonly referred to as heavenly bodies.
The earth, the heavenly bodies and the sky make up the universe.
The universe has stars and a cluster of stars is known as a galaxy or nebula,
with each containing many stars.
Our galaxy in which the solar system exists is the Milky Way.
The solar system
Solar system refers to the grouping of heavenly bodies comprising the
sun and nine planets. The sun is a star around which the planets and
other heavenly bodies The solar system. evolve.

In our solar system, nine planets (Mercury, Venus, Earth, Mars, Jupiter,
Saturn, Uranus, Neptune, and Pluto), over 61 moons, many asteroids

Page | 7
Prefer Calling Sir Obiero Amos @ 0706 851 439 for F1-F4 All Subjects Notes
(mostly in a belt between Mars and Jupiter), comets, meteoroids and other
rocks and gas all orbit the Sun.

Important points to note


➢ A star is a heavenly body possessing its own light which it transmits.

➢ Planets are large and spherical celestial bodies in space which move
around a star such as the sun on their orbits.
➢ An orbit is the path in space which the planet follows as it revolves
around the sun. The orbits of the nine planets are elliptical.
➢ Each planet takes a different length of time to complete one revolution;
because the distance from the sun to each planet varies.
➢ There are other smaller bodies found in the solar system. For example,
between planets mars and Jupiter, there are satellites known as asteroids.
➢ Some planet has their own satellites (moons) that revolve around them

THE PLANETS
1. Mercury
It is the smallest and the nearest planet to the sun, with a distance of about
58 million kilometres from the sun.
It takes approximately 88 earth days to complete one revolution.
It has no satellite.

2. Venus.
Second planet from the sun with an average distance of 108 million
kilometres separating them.
It is one of the brightest planets in the sky and can be seen
with the naked eye. It is structurally similar to the earth
though it is slightly smaller.
It takes 225 earth days to revolve round the sun. It has no satellite.

3. Earth.
The planet on which we live, the only planet that supports life, is the third
from the sun.

Page | 8
Prefer Calling Sir Obiero Amos @ 0706 851 439 for F1-F4 All Subjects Notes
The distance from the sun is 149 million kilometres.
It takes 365.26 days to complete one revolution around the sun.
It has only one satellite, the moon.

4. Mars.
It s fourth from the sun and is slightly smaller than the earth.
Its distance from the sun is 227.9 million kilometres and it takes 687 earth
days to complete one revolution.
It has two moons.
Between it and Jupiter there exist small heavenly bodies called asteroids.

5. Jupiter.
This is the largest planet and with a large number of satellites (sixteen).
It takes 12 earth years to revolve round the sun.
The planet has very thick layers of ice on its surface.
The distance from the sun to Jupiter is 778 million kilometres.

6. Saturn.
It is a unique planet that has a ring around it.
It is the second biggest planet in the solar system.
The distance from the sun is about 1427 million kilometres.
It takes 29½ earth years to complete one revolution.
It has eighteen satellites.

7. Uranus.
The seventh planet from the sun.
It is roughly 4 times bigger than the earth.
The distance from the sun to Uranus is about 2870 million kilometres.
Because of its vast distance from the earth, very little is known about it.
It has one satellite.
It takes 84 earth years to complete one revolution.

Page | 9
Prefer Calling Sir Obiero Amos @ 0706 851 439 for F1-F4 All Subjects Notes
8. Neptune.
The distance from the sun is 4497 million kilometres.
It can only be seen using a powerful telescope.
It takes 165 earth years to complete one revolution.
It has eight satellites.

9. Pluto.
This is the farthest planet.
It is the smallest with an approximate size of 1/6 the size of the earth.
The distance from the sun is 5900 million kilometres.
To complete one revolution round the sun, it takes 248 earth years.
It has one satellite.

OTHER CELESTIAL BODIES.


1. Natural satellites.
A natural satellite is any natural body that orbits around a planet. The solar
system has over 61 satellites, varying in size.
The seven largest, each with a diameter of more than 2500 kilometres are;
a) The earth’s moon

b) The four Galilean satellites of Jupiter (Io, Europa, Ganymede and


Callisto).
c) Titan on Saturn

d) Triton on Neptune.

2. Asteroids.
These are planet-like objects and sometimes known as planetoids, located
between mars and Jupiter. They are believed to be pieces of a planet that
broke up or existing as material that failed to form as planets when the solar
system was forming. They are over 1500 in number and also orbiting around
the sun. Their orbits are so erratic that the constantly collide with each other,
and even with planets.
The erratic orbits are believed a product of Jupiter’s Gravitative pull.

Page | 10
Prefer Calling Sir Obiero Amos @ 0706 851 439 for F1-F4 All Subjects Notes
3. Comets.
A comet is a heavenly body which orbits round the sun. It is thought to be
made up of a ball of ice with dust and frozen gases forming its nucleus. As
the comet nears the sun, the dust points away from the sun, giving the comet
a head and tail morphology. The orbit of a comet crosses the orbits of planets
and at one point it moves so close to the sun.

4. Meteors.
Also known as shooting stars, they refer to a streak of light seen in the sky
on a clear night.
Meteors are a product of meteoroids that burn themselves out( meteoroids are
small bodies of matter which break away from their path in the solar system and enter
the earth’s upper atmosphere at very high speed).
The burning out is due to great friction between them and the atmosphere,
caused by the high speed of entry into the earth’s atmosphere.
The burn up in the regions of between 75 and 115 kilometres above the
surface of the earth.
The much more brilliant meteors produced by larger meteoroids are known
as bolide.

5. Meteorites.
This is a remnant of a meteoroid that failed to burn up completely and
reaches the earth’s surface.They are products of meteoroids that enter the
earth’s atmosphere at low velocity and therefore minimal friction that cannot
cause them to burn up.
Only 25% of their mass reaches the surface as the rest is burned up.They vary
in size from a few grams to 20,000 kg. Sometimes very large meteorites
landing on the ground cause formation of craters.

Page | 11
Prefer Calling Sir Obiero Amos @ 0706 851 439 for F1-F4 All Subjects Notes
THE EARTH.
Origin of the earth and the solar system
The two main theories are;
1. The passing star theory.

The theory was advanced by jeans and Jeffrey’s.


The theory states that the sun existed earlier than the planets. A big star with
greater gravitational pull than that of the sun passed nearby and attracted
large quantities of materials in form of gases from the sun. The materials
split into portions as they cooled and condensed to form planets.
They were then set into orbits around the sun. The smaller materials formed
the heavenly bodies like the moon and asteroids. As cooling continued,
heavier materials collected at the centre and formed the core of the earth.
The less dense materials collected around the core to form the mantle, then
the crust.

2. The nebular cloud hypothesis.


The hypothesis Suggests that the solar system started out as a nebular cloud,
(a large, rotating cloud of dust and gas that looked like an Andromeda spiral). Due
to acceleration in its rotation, Nebula thus flattening into a disk oriented
perpendicularly to its axis of rotation.The planets and sun were concentrated
from the dust and gasses in the cloud by gravitational attraction.
The interior of the earth is believed to be still hot due to
the following reasons; a) Pressure of the overlying
materials generates high temperature.
b) During formation of the earth, the interior cooled slowly that the outer
part, retaining the original temperature.
c) Radioactivity/breaking down of nuclear of atoms release a lot of energy
which generates heat. The shape of the earth.

Page | 12
Prefer Calling Sir Obiero Amos @ 0706 851 439 for F1-F4 All Subjects Notes
The shape of the earth is described as spherical, though not a perfect sphere.
It is slightly wider at the equator and flattened at the poles giving it a shape
called a geoid or oblate spheroid Proofs that the earth is spherical.
a) Circumnavigation. Anybody traveling at constant direction, from any
point on the earth’s surface will eventually come back to the same starting
point.
b) If the earth was flat, the sun would rise and set at the same time over its entire
surface. However, the sun rises east and sets west, with the rays angle
varying from 0 to 90 degrees, proving a curved surface.
c) The horizon of the earth is always circular to an observer. It continues to
expand with increasing height. These are features of a sphere.
d) In lunar eclipses, the earth’s shadow on the moon is always seen to be circular
and the only object which casts a circular shadow is a sphere.
e) All other planets of the solar system are spherical. Since the earth is one
of the planets, it may be assumed to be, like the rest, spherical in shape.
f) Examining photographs taken in space from rockets at very high latitude-
320km and more- shows the earth’s horizon as a curved line.

The size of the earth.


The latest calculation and satellite observations give the following
mathematical data for planet earth:
• Approximate distance from the sun-149 560 000kilometres.

• Polar circumference-39995 kilometres.

• Equatorial diameter- 12756kilometres. Inclination of the equator from


the horizon-23˚ Length of day-24 hours.
• Length of year-365.26 days.
• Surface area of the earth-510 x 10 sq. kilometres. 71% comprise water
surface.

Page | 13
Prefer Calling Sir Obiero Amos @ 0706 851 439 for F1-F4 All Subjects Notes
MOVEMENT OF THE EARTH
There are two types of earth movements;
a) Earth rotation.
b) Revolution.

EARTH’S ROTATION

The term Earth rotation refers to the spinning of our planet on its axis. One rotation ta kes exactly twenty -

four hours and is called a day.


Rotation of the earth

Through 360˚, the earth takes 24 hours to make one rotation. This implies

that for every 15˚ of rotation, it takes one hour or for every four minutes,

it takes 1˚

EFFECTS OF ROTATION.

1. Day and night. The Earth’s rotation is responsible for the daily cycles of
day and night.

DAY AND NIGHT- ILLUSTRATION

imp ortan t to all levels of KCSE

curriculum@2011 137
important to all levels of KCSE

Page | 14
Prefer Calling Sir Obiero Amos @ 0706 851 439 for F1-F4 All Subjects Notes
2. High tide and low tide. A tide is a rhythmic rise and fall of sea level
caused by the gravitational forces of the moon and the sun upon the
rotating earth.
LT2

Moon ’ s pull of gravity


HT2 HT1
EARTH MOON

LT1

3. Deflection of winds and ocean currents. As the earth rotates from west
to east, winds and ocean currents are deflected. In the southern
hemisphere, they are deflected to the left. In the north, they are deflected
to the right.

4. Time difference between longitudes. The rotation of the earth causes a


difference in time of 1 hour in every 15˚ interval between longitudes.
This is because the earth takes 24 hours to go through 360˚. Through
1˚the earth takes 4 minutes. Places on same longitude record same time
known as local time. Local time at Greenwich Meridian (longitude 0˚)
is called the Greenwich Mean Time.
Any movement westwards from GMT witnesses a loss of 4 minutes in
every 1˚. Towards the east, one gains 4 minutes in every 1˚

Calculation of time using longitude.


➢ Time calculation involves finding the difference in degrees of the
longitudes of the two given points.
➢ This difference is then multiplied by 4 minutes, the time taken by the earth
to rotate through 1˚.

Page | 15
Prefer Calling Sir Obiero Amos @ 0706 851 439 for F1-F4 All Subjects Notes
➢ Determine if the place whose time you are finding is found on the east or
on the west of Greenwich Meridian. If on the east, add the time to the
given time. If on the west, subtract.

Standard time and time zones.


Time zones are the internationally agreed divisions of the world into zones,
each approximately 15˚wide with regular intervals across the oceans and
irregular ones over land.
The irregularities over land are to avoid splitting any one country into
different time zones as this could create confusion. The world has 24 time
zones. The time recorded by countries in the same time zone is called
standard time.
The world time zone map.
World time zones

important to all levels of KCSE


curriculum@2011 144

International Date Line.


This is the line following approximately longitude 180˚ except where it has
to cross through a group of islands or a country. Where it curves, the aim is
to avoid splitting a country or islands belonging to one country into different
time zones. On crossing this line, one loss or gains a day.

Page | 16
Prefer Calling Sir Obiero Amos @ 0706 851 439 for F1-F4 All Subjects Notes
REVOLUTION.
The orbiting of the Earth around the Sun is called a revolution and it takes
365.26 days to complete one cycle. A normal year is therefore 365 days while
a leap year (every fourth year) has 366 days.
When the Earth is revolving, its axis is not at right angles to this surface, but
inclined at an angle of about 23.5° from the perpendicular.
The results of the movement of the earth around the sun and inclination of
its axis are;
1) It causes the seasons, by controlling the intensity and duration of sunlight
received by locations on the Earth.
The four resultant seasons are spring, summer, autumn and winter.
The four s easons
nd
Dec. 22

rd
Sept. 23

March 21st

st
important to all levels of KCSE

June 21st

The seasons are mainly experienced in the high and mid latitudes. When the
sun is overhead at the equator on March 21st, it is the beginning of spring in
the temperate regions of the northern hemisphere when snow melts and
vegetation begins to blossom and animals spring to life. It is autumn at the
same time in the mid latitudes of the southern hemisphere.
Winter is the season when the temperatures are very low.

Page | 17
Prefer Calling Sir Obiero Amos @ 0706 851 439 for F1-F4 All Subjects Notes
2) It results in the solstices and equinoxes. .
On June 21st and December 22nd, the sun’s overhead position is over the
tropic of cancer and Capricorn respectively. This time of the year is known
as solstice. During the summer solstice (June 21st) the Earth's North Pole is
tilted 23.50 towards the Sun. all places above latitude 66.5 0 N in 24 hours of
sunlight, while locations below latitude of 66.50 S are in darkness.
During the December 22nd solstice (winter solstice), The North Pole is tilted
23.50 away from the Sun. On this date, all places above latitude 66.5 0 N are
now in darkness, while locations below latitude 66.50 S receive 24 hours of
day.
During the equinoxes, the axis of the Earth is not tilted toward or away
from the Sun. (On September 23rd and March 21st). Day lengths on both of
these days, regardless of latitude, are exactly 12 hours.

3) Varying length of day and night


The axis of the earth is inclined to its ecliptic plane at a certain angle.
If the axis of the earth was perpendicular to the elliptical plane, all parts of
the earth would receive equal nights and days at all times of the year.
However, variations are witnessed governed by the apparent position of the
overhead sun.
For example in December, the north experiences longer nights than the south
because the sun is overhead in the south.

4) Changes in position of midday sun.


The annual change in the relative position of the Earth's axis in relationship
to the Sun causes the height of the Sun or solar altitude to vary in our skies.
The structure of the earth.
Sima ( inner
Crust)
Sial ( outer
A
Crust)

Atmosphere
( mixture of gases )
Mantle Outer
( asthenosphere ) core
Inner
core

Page | 18
Prefer Calling Sir Obiero Amos @ 0706 851 439 for F1-F4 All Subjects Notes
Earth's radius is about 6,371 km and the radius of the core is about 3,486 km
(the inner core radius is about 1,217 km.
The earth is made up of three main layers namely; the crust, mantle, and
core. Other layers ate the Atmosphere which is the layer of gases
surrounding the earth and the hydrosphere which is the layer of water
covering the earth.
1. Earth’s Crust:

The crust ranges from 16–24 km in thickness though it may extend upto
80km here high mountains exist and reduce upto 6km thick where deep
ocean trenches exist.. It is the outermost layer.
The two subdivisions of the crust are;
a) The thin inner oceanic crust. It mainly consists of silica and magnesium;
it is therefore called sima (si-silica and ma-magnesium). It has an
average density of 2.7 gms/cc.
b) The thicker continental crust. The main mineral constituents of the
continental mass are silica and alumina; it is thus called sial (si-silica,
65–75% and al-alumina). Its density is between 2.8 and 3.0 gms/cc.
NB; the density of the rocks of the earth appears to increase with increasing
depth. The uppermost mantle together with the crust constitutes the
lithosphere. The crust-mantle zone of discontinuity is called the Mohorovicic
discontinuity or Moho.

2. Mantle
Earth's Mantle extends to a depth of 2,890 km, and is the thickest layer of the
Earth. The mantle is composed mainly of olivine-rich rock.
The temperature of the mantle increases with depth. The average density of
the mantle is 3.0gms/cc – 3.3gms/cc.
Rocks in the upper mantle (region of the mantle within 1000km of the
surface) are cool and brittle enough to break under stress, while rocks in the
lower mantle are hot and soft (but not molten) and flow when subjected to
forces instead of breaking. Most of the heating in the mantle is due to
radioactive decay. The zone of discontinuity between the mantle and the
core is called Guttenberg discontinuity.

Page | 19
Prefer Calling Sir Obiero Amos @ 0706 851 439 for F1-F4 All Subjects Notes
3. Core
Earth's Core is thought to be composed mainly of an iron (80%), and nickel
alloy. Seismic measurements show that the core is divided into two parts;
a) A liquid outer core extending to a radius of ~3,400 km. the
temperatures there are adequate to melt the iron-nickel alloy.it
surrounds the inner core.
b) A solid inner core with a radius of ~1,220 km. though its temperature
is higher than the outer core, tremendous pressure, produced by the
weight of the overlying rocks is strong enough to crowd the atoms
tightly together and prevents the liquid state. the inner core rotates
slightly faster than the rest of the planet

WEATHER
DEFINITION.
It is the daily atmosphere conditions of a place for a short period of time.
There are several elements that are used in determining the condition of the
atmosphere. They are:
➢ Temperature

➢ Wind

➢ Sunshine

➢ Humidity

➢ Precipitation

➢ Cloud development and cover ➢ Atmospheric


pressure.
How does climate differ from weather?
Weather is the current atmospheric conditions, including temperature,
rainfall, wind, and humidity at a given place.
Climate, on the other hand, is the general weather conditions over a long
period of time.
Weather data such as temperature variations and precipitation rates for the
past 30 years are used to compile an area's "average" weather. Weather
Stations

Page | 20
Prefer Calling Sir Obiero Amos @ 0706 851 439 for F1-F4 All Subjects Notes
These are locations that are carefully chosen so that accurate and reliable
data on weather elements are collected, observed, measured and recorded
from there.
Weather stations must be clear of obstructions that could block the wind or
cast shadows. They are fenced off so that animals cannot get in and damage
the equipment.
The weather instruments are found on land in weather stations.
An important part of a weather station is a Stevenson Screen, which holds
the thermometers.

60 cm
The Stevenson screen
40cm
Insulated roof
This is a white box with slatted/louvered sides. The
Sides are made of white colour reflects the direct sun’s rays and
double wooden
65 cm
louvres. One side is
hinged and it acts as a slats/louvers on the sides allow air to pass through
door
freely. The box is raised on stilts/stands so that it is
easier to read the instruments and there is no effect
stand
121cm
from the ground. Accurate temperature and
humidity readings can then be taken.

Instruments placed outside the Stevenson screen


1. The Raingauge to measure amount of precipitation/rainfall.

2. The weather vane shows the direction.

3. The anemometer allows the wind speed to be measured.

4. A Campbell stokes sunshine recorder measures the duration of


sunshine.
5. The evaporimeter that measures the rate and amount of evaporation.

Page | 21
Prefer Calling Sir Obiero Amos @ 0706 851 439 for F1-F4 All Subjects Notes
Inside the screen we find the rest of the instruments which are thermometers
1. The wet and dry thermometers, (also called hygrometers) allow the
calculation of the humidity of the air.
2. The maximum and minimum allow the 24 hour temperatures to be
recorded. This is sometimes called a 'Six’s thermometer.
3. The mercury barometer and an aneroid barometer to measure

pressure.

The Stevenson screen serves two purposes;


a) To ensure the delicate instruments kept in it are safe.

b) To provide the shade conditions required for accurate temperature


readings.

The weather
elements.

A. SUNSHINE.
This Refers to the direct rays of sunlight reaching the surface of the earth.
Sunshine duration and intensity depends on factors such as latitude, aspect
and cloud cover.
Places of the same sunshine intensity on a map are joined by lines referred
to as Isohels.

B. TEMPERATURE.
This is the degree of sensible heat within the atmosphere.
It is a Measure of hotness expressed in terms of any of several arbitrary
scales, such as Fahrenheit, Celsius, or Kelvin.
The sun is the source of heat energy that maintains the temperature of the
earth’s surface and atmosphere. The sun’s energy is transmitted in the forms
of short-wave rays, a process called insolation or solar radiation.
Only a small fraction of solar radiation reaches the earth’s surface. Why?
What factors determine the amount of solar radiation reaching the surface?

Page | 22
Prefer Calling Sir Obiero Amos @ 0706 851 439 for F1-F4 All Subjects Notes
1. The intensity of the sun’s radiation in the space and the earth’s average
distance from the sun.
2. The transparency of the atmosphere i.e. transmission, absorption,
scattering and reflection of the sun’s rays by particles found in the
atmosphere.
3. The position of the earth on its orbit which produces different seasons.
This causes insolation to vary with the time of the day and period of
the year.
4. The inclination or angle of the surface on which the sun’s ray fall.

5. The area and nature of the surface on which the rays fall.

Lines drawn on a map showing places with the same temperatures are called

Isotherms.
The atmosphere is heated through three main processes;
~ Radiation
~Conduction.
~ Convection.

1. Radiation.
This refers to the transfer of energy via electromagnetic waves.
Examples: o sun warms
your face o
apparent heat of a
fire
Radiation travels in waves that are very small and are transmitted from one
place to another without the help of a medium (travel in a vacuum).
The earth emits heat by long-wave radiation called terrestrial radiation
which is invisible.
The earth radiates its maximum heat at a much longer wavelength than the
sun.

Page | 23
Prefer Calling Sir Obiero Amos @ 0706 851 439 for F1-F4 All Subjects Notes
Difference between solar and terrestrial radiation
~ Solar radiation is short wave while terrestrial
radiation is long wave. ~ Solar radiation is
partially visible as sunlight ~ Solar radiation
takes place only daytime.

2. Conduction.
It is the transfer of heat through matter, without moving it (matter). It is the
transfer of heat energy through motion, from one molecule to another.
Air in the atmosphere is heated by direct contact with the earth.

Illustration of conduction, radiation and convection.

3. Convection.
This is the transfer of heat through mass movement of a substance. The
"substance" could be air or water. It takes place when air is heated by the
earth, becomes less dense, and then rises to higher levels transferring heat
Page | 24
Prefer Calling Sir Obiero Amos @ 0706 851 439 for F1-F4 All Subjects Notes
with it. Cold air above the surface descends down to replace warm rising air.
This movement of lighter air upwards and dense air down wards creates
cycles called convectional currents.

Factors influencing Air temperature.


The distribution of temperature over the earth surface depends on following
factors:
1. Latitude:
Highest temperatures are generally at the equator and the lowest at the
poles. This is because the Sun shines all the year round for 12 hours out of 24
hours at the Equator. At the poles it is only 6 hours for six months. Also, As
Earth has a spherical shape, the angle of Sun’s rays on surface of the Earth
differs from one place to another and temperature changes at every latitude.

2. Altitude or Height above Sea Level


Temperature decreases with height in troposphere at a rate of 0.6 C/100cm.
This is known as normal lapse rate. First, the Sun’s rays heat the earth’s
surface then, the lower atmosphere is heated by conduction from the earth.
The temperature nearer the earth’s surface is more than that higher altitude.
Mt. Kenya is on the Equator Yet its peak is always covered under snow,
because it is over 5000 metres above sea level.

3. Length of day
The longer the period of solar insolation, the greater the quantity of radiation
received at a given place on the earth surface. The greater the quantity, the
more the heat that will be generated by the earth.

4. Winds:
Various types of wind affect temperature. Winds are a medium of transfer
of heat from one place to another. Winds carrying warm air bring a warming
effect to a place.

Page | 25
Prefer Calling Sir Obiero Amos @ 0706 851 439 for F1-F4 All Subjects Notes
5. Ocean currents:
Ocean currents may be warm or cold. When the winds blow over them, they
get their warmth or cold and affect the temperatures of coastal areas. For
example, the warm North Atlantic Drift raises the winter temperature of
North-West Europe.

6. Aspect.
It is the direction in which a slope faces. The effect is greater at higher
latitudes than in tropical regions where slopes receive more or less equal
hours of the day and night.
Slopes facing north in the southern hemisphere are warmer than
the ones facing south.

7. Cloud cover.
The presence of clouds in the sky prevents the amount of solar radiation
coming to the earth’s surface. It also prevents the ground radiation, leaving
the earth’s surface. In the equatorial areas, because of the abundance of cloud
cover, day temperatures are not so high and night temperatures not so low.

8. Distance from the sea.


Along same latitude, the amount of solar insolation received is expected to
be the same. However, there is often considerable difference between
temperatures of land and water surfaces. Why?
a) The specific heat of water is greater than of the land. To raise the
temperature of water and of land by 1 C, water requires five times
as much heat as land.
b) The water surface reflects more of the sun’s rays back into the
atmosphere than does the land surface. Water takes longer than land
to absorb the same amount of solar energy.
c) The transparent nature of water allows the sun’s rays to penetrate to
grater depths, so more heat is retained in the water.
Summer temperatures in coastal areas are lower than in the continental
interiors.

Page | 26
Prefer Calling Sir Obiero Amos @ 0706 851 439 for F1-F4 All Subjects Notes
C. HUMIDITY.
Humidity is a measure of the amount of (invisible) water vapour or the
degree of dampness in the atmosphere. It is the condensation of this vapour
which gives rise to clouds, rain, snow, dew, frost and fog. When air contains
the maximum amount of vapour possible for a particular temperature, it is
said to be saturated. Usually the air is not saturated, containing only a
fraction of the possible water vapour.
The amount of vapour in the air can be expressed in a variety of ways.
a) Absolute humidity indicates the actual amount of water vapour present
in a sample of air, or the vapour concentration, the mass of water
vapour in a given quantity of air. 1kg of air might hold up to 25 grams
of water vapour in the tropics, but almost nothing in cold Polar
Regions.
b) Relative humidity is the ratio of the actual mass of vapour in the air to
the mass of vapour in saturated air at the same temperature. For
example, air at 10°C contains 9.4 g/m3 of water vapour when
saturated. If air at this temperature contains only 4.7 g/m3 of water
vapour, then the relative humidity is 50%.

Factors influencing humidity


1. Temperature. When temperature rises, relative humidity is lowered if the
amount of moisture remains constant. When air is heated, it will expand
leading to increased volume. Cooled air decreases in volume. Thus space
occupied by moisture, becomes less.
2. Air pressure. When air is compressed, it will warm up and its density
becomes lower. Its ability to absorb more moisture increases.
3. Supply of moisture. This is determined by distance from the sea or large
water body. Increased supply of moisture leads to more water vapour in
the air (absolute humidity). If the temperature of that air increase, it will
absorb even more moisture.
4. Latitude. Humidity is higher in low latitudes than in high latitudes due
to greater rate of evaporation in the lower latitudes. The warmer
temperatures cause the air to expand and therefore increase its capacity
of holding moisture.
Page | 27
Prefer Calling Sir Obiero Amos @ 0706 851 439 for F1-F4 All Subjects Notes
5. Seasons. Also the amount of moisture in the air is higher in the summer
than in winter and during the day than at night.

D. CLOUDS
A cloud is a mass of tiny visible particles of water or ice, formed by
condensation and suspended in the atmosphere. Condensation is the change
of water from its gaseous form (water vapour) into liquid water.
Condensation generally occurs in the atmosphere when warm moist air rises
to reach the condensation level, cools and loses its capacity to hold water
vapour. As a result, excess water vapor condenses to form cloud droplets..
If moist air rises to an altitude where temperature is below 0 C, the
condensed water droplets freeze to form ice particles. The clouds at this
level may be made entirely of ice crystals or ice crystals mixed with droplets
of supercooled water.
Super cooling is a situation where water remains in a liquid state at
temperatures below freezing point. This may be caused by lack of sufficient
particles (freezing nuclei) on which ice particles can form. Salt particles from
the oceans are very effective and are reffered to as hygroscopic nuclei.
Condensation can also be caused by advection (the horizontal transport or
transfer of a quality such as heat and cold from one point to another).This results
in formation of advection fog. Common cloud classifications

Page | 28
Prefer Calling Sir Obiero Amos @ 0706 851 439 for F1-F4 All Subjects Notes
Clouds are classified according to their appearance, height and form.

1. High-Level Clouds
High-level clouds form above 6,000m and since the temperatures are so cold
at such high elevations, these clouds are primarily composed of ice crystals.
Examples.
a) Cirrus Clouds. The clouds are thin and often wispy and exist at heights
greater than 20,000 feet. Cirrus generally occurs in fair weather and point
in the direction of air movement at their elevation.
b) Cirrostratus Clouds These are sheet-like, high-level clouds sometimes
covering the entire sky and composed of ice crystals. They are relatively
transparent, as the sun or the moon can easily be seen through them.
Sometimes the only indication of their presence is given by an observed
halo around the sun or moon.
c) Cirrocumulus clouds. They are white clouds consisting of white crystals.
They have a thin base, and are either globular in appearance or arranged
in ripples.

2. Mid-Level Clouds.
The bases of mid-level clouds typically appear between 2,000 to 6,000 m.
Because of their lower altitudes, they are composed primarily of water
droplets; however, they can also be composed of ice crystals when
temperatures are cold enough. Examples.
a) Altocumulus Clouds. They may appear as parallel bands or rounded
masses. The presence of altocumulus clouds on a warm and humid
summer morning is commonly followed by thunderstorms later in the
day.
b) Altostratus clouds. Often confused with high-level cirrostratus, however, a
halo is not observed around the sun (or moon) when viewed through
altostratus. The sun (or moon) is only vaguely visible through altostratus
clouds and appears as if it were shining through frosted glass.

Page | 29
Prefer Calling Sir Obiero Amos @ 0706 851 439 for F1-F4 All Subjects Notes
3. Low-level Clouds
Low clouds are of mostly composed of water droplets since their bases
generally lie below 2,000 meters.
Examples.
a) Nimbostratus Clouds. Nimbostratus is dark, low-level cloud accompanied
by light to moderately falling precipitation. Because of the fog and falling
precipitation commonly found in nimbostratus clouds, the cloud base is
typically very diffuse and difficult to accurately determine.
b) Stratocumulus Clouds. They generally appear as a low, lumpy layer of
clouds that is sometimes accompanied by weak intensity precipitation.
Stratocumulus vary in color from dark gray to light gray and may appear
as rounded masses, rolls, etc., with breaks of clear sky in between.
c) Cumulus Clouds. Cumulus has the appearance of floating cotton and has
a lifetime of 5-40 minutes. Known for their flat bases and distinct outlines,
cumulus exhibit only slight vertical growth, with the cloud tops
designating the limit of the rising air.
d) Stratus. These are clouds that form a uniform grey layer that resembles
fog and has a low base.
e) Nimbostratus. It is a rain cloud. It is shapeless, dark-grey and dense. It
spreads all over the sky in uniform layers.

4. Vertically Developed Clouds.


These clouds can grow to heights in excess of 39,000 feet (12,000 meters),
releasing incredible amounts of energy through the condensation of water
vapor within the cloud itself. Example.
a) Cumulonimbus Clouds .Cumulonimbus clouds are much larger and more
vertically developed than cumulus. Lower levels of cumulonimbus
clouds consist mostly of water droplets while at higher elevations, where
temperatures are well below 00C, ice crystals dominate.
Lines drawn on a map passing through places having the same amount of
cloud cover are known as isonephs.

Page | 30
Prefer Calling Sir Obiero Amos @ 0706 851 439 for F1-F4 All Subjects Notes
E. PRECIPITATION.
Precipitation is the deposition of products of the condensation in the
atmosphere under gravity onto the earth’s surface. It occurs when a local
portion of the atmosphere becomes saturated with water vapour.
Precipitation forms as smaller droplets coalesce via collision with other rain
drops or ice crystals within a cloud. The main forms of precipitation include
drizzle, rain, sleet, dew, frost, mist and fog, snow and hail.

a) Dew.
Dew is a type of precipitation where water droplets form on the ground, or
on objects near the ground in a process called condensation of moisture. Dew
forms when the ground surface and other exposed objects, such as tips of
grass or leaves, lose heat by radiation to the sky.
The dew point is the critical temperature at which air being cooled becomes
saturated with water vapour.
Dew forms under the following conditions.
(i) The air should be calm so that it can remain in contact with the ground
long enough to be cooled below dew point.
(ii) Daytime should be warm to accelerate evaporation and thus provide a
lot of water vapour in the air.
(iii) A cloudless night accelerates the rate at which the earth loses the heat
gained during the day.

b) Hail
This is a form of snow consisting of roughly spherical lumps of ice.
Hailstones can grow beyond 5mm in diameter and weigh more than 2.5 km.
The formation of hailstones occurs when ice crystals falling through the
deep and active cumulonimbus clouds encounter supercooled cloud
droplets which mould themselves around the ice crystals before freezing to
cause the additional layer of clear ice to the soft hailstone. During the
process, very strong up currents must be present to hold the hailstones up
in the cloud as they grow.

Page | 31
Prefer Calling Sir Obiero Amos @ 0706 851 439 for F1-F4 All Subjects Notes
c) Hoar frosts.
These are very tiny ice crystals which are formed when water vapour in the
atmosphere is converted into ice crystals without first becoming liquid. The
crystals are formed on vegetation or any object that has been chilled below
freezing point by radiation cooling. This deposit of ice crystals may
sometimes be so thick that it might look like snow.

d) Rime.
Rime is a white or milky and opaque granular deposit of ice formed when
fog is driven by wind and deposited on objects whose temperatures are
below freezing point. Such objects include telephone posts, power lines and
on top of houses. Rime is denser and harder than hoar frost and is common
in the midlatitudes. When these ice crystals join together, they are
popularly known as frost feathers.

e) Snow.
This is the precipitation that falls in the form of flakes of numerous tiny ice
crystals. Snow crystals form when tiny supercooled cloud droplets freeze.
Once a droplet has frozen, it grows in the supersaturated environment.
These large crystals may fall through the atmosphere due to their mass,
and may collide and stick together in clusters, or aggregates to form
snowflakes.

f) Sleet.
This is a form of precipitation which is a mixture of rain and snow or
partially melted snow that falls on the surface of the earth. Sleet occurs
when the surface of the earth is at or just above freezing point- 0 C

g) Thunderstorms.
This is the precipitation accompanied by thunder and lighting. Thunder is
the explosion or bomb like sound which occurs in the sky where there are
thick cumulonimbus clouds. Thunderstorms form when moist, unstable air
is lifted vertically into the atmosphere. Lifting of this air results in
condensation and the release of latent heat.

Page | 32
Prefer Calling Sir Obiero Amos @ 0706 851 439 for F1-F4 All Subjects Notes
The process to initiate vertical lifting can be caused by:
1. Unequal warming of the surface of the Earth.

2. Orographic lifting due to topographic obstruction of air flow.

3. Dynamic lifting because of the presence of a frontal zone.

Immediately after lifting begins, the rising parcel of warm moist air begins
to cool because of adiabatic expansion. At a certain elevation the dew point
is reached resulting in condensation. With the vertical extension of the air
parcel, the cumulus cloud grows into a cumulonimbus cloud. Severe
weather associated with some these clouds includes hail, strong winds,
thunder, lightning, intense rain, and tornadoes.

During the cumulus stage, strong updrafts act to build the storm. The
mature stage is marked by heavy precipitation and cool downdrafts in
part of the storm. When the warm updrafts disappear completely,
precipitation becomes light and the cloud begins to evaporate.

h) Fog and mist.


Fog is made of tiny liquid water droplets; light weighted enough to remain
suspended in the air. The term fog is used when visibility reduces to less
than 1km, Whereas mist is reported when visibility exceeds 1km.
The five most important factors for fog formation are;
~ long nights during colder months;
~ clear skies and light winds;
~ moist air;
~ a low-level temperature inversion
~ Sufficient supply of condensation nuclei.

Types of fog are:


1. Radiation or ground fog - in mountain areas and cold pools often seen as
valley fog. Radiation fog might be rather shallow over damp and wet
ground after a rainy night and is sometimes called precipitation fog.
Radiation fog might also form as a low stratus cloud forming just

Page | 33
Prefer Calling Sir Obiero Amos @ 0706 851 439 for F1-F4 All Subjects Notes
underneath an inversion, gradually expanding and descending towards
the ground surface.
2. Advection fog - is also called sea fog, when warm air flows over relatively
cold sea surface, but is just as common over land. Advection fog is often
associated with the passage of cold and warm fronts and thus is also
known as frontal fog.
3. Hill fog. A type of advection fog which forms as a low sheet cloud in the
hills as result of moist air stream advancing inland from the sea.
4. Frontal fog. This forms at points where cold air masses meet with warm
air masses. The cold air causes moisture in the warm air to condense and
form fog.
5. Steam fog. This is a type of fog which forms when cold air passes over the
surface of a much warmer fresh water body. As the warm water is cooled,
it appears to be steaming as the condensing water vapour above it forms
fog
6. Ice fog. Formed when water vapour is converted directly into ice crystals.
When it is formed over salty seas, it is called Arctic smoke.

i) RAINFALL
This is a form of precipitation falling from the clouds in the form of drops of
water. When water droplets become too heavy to be suspended in the
atmosphere (in a given cloud) they fall as rainfall or drizzles. A drizzle is a
uniform type of rain comprising tiny drops of liquid water and usually
falling from layers of low clouds. A drizzle has less intensity when compared
to rain.
Rain consists of larger raindrops falling at a uniform rate and lasting for a
long time. It falls from deep layers of clouds especially Nimbostratus and
altostratus clouds.
Rain showers are composed of relatively large drops compared to those of
rain and drizzle but the drops are rather scattered. Showers start and end
suddenly and can be more intense than rain, falling from cumulus and
cumulonimbus clouds.
Lines drawn on a map to show areas receiving the same amount of rainfall
are called isohyets.
Page | 34
Prefer Calling Sir Obiero Amos @ 0706 851 439 for F1-F4 All Subjects Notes
Rainfall forms in the falling ways;
a) When wind blow over a mountainous region.

b) When hot air rises by convectional currents.

c) When warm air rises over cold air.

TYPES OF RAINFALL
Relief or Orographic rainfall
Relief rain occurs in areas where onshore winds rise over hilly or
mountainous regions lying regions lying parallel to the coast.

Orographic precipitation
Orographic precipitation occurs on the windward side of mountains
and is caused by a large-scale flow of moist air across the mountain
ridge, resulting in adiabatic cooling and condensation. Moisture is
removed by Orographic lift, leaving drier air descending and generally
warming, on leeward side where rains shadow is observed.
Relief rainfall usually falls in light showers over a long period of time. It
rarely falls in the lowland areas.

Page | 35
Prefer Calling Sir Obiero Amos @ 0706 851 439 for F1-F4 All Subjects Notes
Convection
The rain is caused by hot air expanding and rising. The air cools as it rises
and finally cond4snses to clouds high up in the atmosphere.
If the air cools beyond condensation level, raindrops are formed and rain
falls.
Convective rain occurs from convective clouds, e.g., cumulonimbus. It falls
as showers with rapidly changing intensity.

Convection precipitation.
Convective precipitation falls over a certain area for a relatively short time,
as convective clouds have limited horizontal extent.
Thunderstorms, lighting and Hail indicates convection. The rain is torrential
falling in large drops and lasting a short time usually 15-20 minutes.
In Kenya the rain falls in the lake region.

Cyclonic rain/ frontal rain.


In the mid-latitudes, moist warm wind from the tropics and cold dry winds
from the Polar Regions may converge.
The cold polar winds, being heavier will sink below the warm lighter
tropical winds.
The Moisture in the tropical warm winds will condense. The resultant
rainfall is known as cyclonic or frontal rain.

Page | 36
Prefer Calling Sir Obiero Amos @ 0706 851 439 for F1-F4 All Subjects Notes
Cyclonic precipitation.
The point where the two air masses meet is called a frontal zone.

F. ATMOSPHERIC PRESSURE
Atmospheric pressure is defined as the force per unit area exerted against a
surface by the weight of the air above that surface.
Pressure decreases with increase in altitude at the rate of 1 millibar for every
10 metres.
Standard sea-level pressure 1013mb.
Near the Earth’s surface the pressure decreases with height at a rate of about
3.5 millibars for every 30 metres (100 feet).

Factors that affect atmospheric pressure


1. Altitude (or height above sea level).
Air density decreases with height away from the surface of the earth
(because the pull of the earth's gravity is less). The fewer number of gas
molecules at higher altitudes means fewer molecular collisions and a
decrease in air pressure.

Page | 37
Prefer Calling Sir Obiero Amos @ 0706 851 439 for F1-F4 All Subjects Notes
2. Temperature.
When atmospheric air is heated (such as by radiation from the sun), the air
molecules become more active, which increases the space between them and
reduces the air density.
Lowering the air density decreases the amount of pressure exerted by the
air. Warm air is less dense than cold air and exerts less pressure.

3. Earth Rotation
Variations in the speed of rotation once in 24hours of regions on the earth’s
surface leads to differential expansion of air between the equator and polar
regions. At the equator there is more expansion space for air due to the high
speed of rotation. Air blowing from the poles towards the equator crosses
latitudes that are getting longer. It therefore spreads out to occupy more
space making its pressure to decrease (temperate low pressure belt-60°N and
60°S).

WORLD DISTRIBUTION OF PRESSURE


If the earth’s surface were uniform, the pressure pattern would be as follows.
1) The Equatorial Low Pressure zones. The belt around the equator that is a
zone of light and indeterminate winds called doldrums or equatorial
trough. It is a zone of high temperature and high humidity. Trade winds
converge here forming the I.T.C.Z
2) The sub-tropical high pressure belt. It is found in regions of latitudes 30 N
and S of the equator. It is a region of calm and descending air. It is the
source of trade winds. And the westerlies.
3) The temperate low pressure zone. It is also known as the sub-tropical low
pressure belt and is the meeting [point of the westerlies and the polar
easterlies. It exists in regions 60°N and 60°S of the equator. The low
pressure is due to the earth’s rotation.
4) The polar High pressure. It exists over the N and S poles. It is characterized
by currents of descending calm air of low temperatures. It is the source
of polar easterlies

Page | 38
Prefer Calling Sir Obiero Amos @ 0706 851 439 for F1-F4 All Subjects Notes
NB- the LP belts are not permanent in their positions, shifting constantly
with the apparent path of the overhead sun. They are also not continuous
as depicted in the diagram due to the irregular distribution of land and
water masses. In continental interiors, HP in winter and LP in summer
are created leading to the formation of pressure cells.
World pressure belts.
THE PRESSURE SYSTEMS
These are associated with the development of pressure cells and the resultant
air masses.
An air mass is a large body of air that has similar temperature and moisture
properties throughout and covering an extensive area.
The best source regions for air masses are large flat areas where air can be
stagnant long enough to take on the characteristics of the surface below.

The four main Air masses and their sources


1. Arctic and Antarctic Airmass. The air mass has extremely cold
temperatures and very little moisture. Originating over the Arctic
Ocean and the ice sheets of Greenland in winter.

Page | 39
Prefer Calling Sir Obiero Amos @ 0706 851 439 for F1-F4 All Subjects Notes
2. Polar Air mass. It is cold and dry, originating from high latitudes,
typically as air flowing out of the polar high.
3. Tropical airmass. It is Hot and very dry, originating from the arid and
desert regions in the subtropical high pressure belt during summer.
4. The Equatorial airmass. It forms over the equatorial oceans. It is hot and
unstable.

Air masses that form over oceans are known as maritime air masses and are
usually moist. Those that form over the land are continental air masses and
are thus dry.
When two air masses of different characteristics meet, a front develops.
There are two pressure systems in the world.
• The cyclones.

• The anticyclones.

1. Cyclone- Also low pressure systems.


A cyclone is a low pressure system, characterized by amass of air in which
isobars form an oval shape with the lowest pressure registered at the centre.
The wind flow around low pressure (cyclonic) systems is counterclockwise
in the Northern hemisphere and clockwise in the Southern hemisphere. This
is a consequence of the Coriolis force.

Page | 40
Prefer Calling Sir Obiero Amos @ 0706 851 439 for F1-F4 All Subjects Notes
Low pressure systems in the Northern hemisphere.
They exist in two types;
1. Depressions.

2. Tropical cyclones.

1. Depressions/ Mid-Latitude Cyclones/frontal cyclones.


These are large traveling atmospheric cyclonic storms up to 2000 kilometers
in diameter with centers of low atmospheric pressure.
Depressions are the dominant weather event of the Earth’s temperate
latitudes forming along the polar front in the belt of the westerly winds.
They are the result of the dynamic interaction of warm moist westerlies and
cold polar air masses at the polar front.

This interaction generates friction which causes the warm air (squeezed into
a wedge called a warm sector) to be cyclonically lifted vertically into the
atmosphere where it combines with colder upper atmosphere air.
Around the low, winds blow anti-clockwise and inwards (clockwise and
inward in the Southern Hemisphere). West of the low, cold air traveling
from the north and northwest creates a cold front extending from the
cyclone's center to the southwest. Southeast of the low, northward moving
warm air from the subtropics produces a warm front. Precipitation is
located at the center of the low and along the fronts where air is being
uplifted.
The mid-latitude cyclone is rarely motionless and commonly travels about
1200 kilometers in one day. Its direction of movement is generally eastward.

Page | 41
Prefer Calling Sir Obiero Amos @ 0706 851 439 for F1-F4 All Subjects Notes
Typical paths of mid-latitude cyclones are represented by black arrows. This
image also shows the typical paths traveled by subtropical hurricanes (green
arrows).

Fronts, winds patterns, pressure patterns, and precipitation distribution found in an


idealized mature mid-latitude cyclone.

In this cross-section, we can see how air temperature changes as we move from behind
the cold front to a position ahead of the warm front. Behind the surface position of the
cold front, forward moving cold dense air causes the uplift of the warm lighter air in
advance of the front to form cumulus and then cumulonimbus clouds which produce
heavy precipitation and thunderstorms.
Along the gently sloping warm front, the lifting of moist air produces first
nimbostratus clouds followed by altostratus and cirrostratus.

Page | 42
Prefer Calling Sir Obiero Amos @ 0706 851 439 for F1-F4 All Subjects Notes
Vertical cross-section of the line A-B in Figure above.

During Frontal cyclone development, Outflow results in the development of


an upper air vacuum. To compensate for the vacuum in the upper
atmosphere, surface air flows cyclonically upward into the outflow to
replenish lost mass. The process stops and the mid-latitude cyclone fade
away when the upper air vacuum is filled with surface air.
Mid-latitude cyclones cause far less damage than tropical cyclones or
hurricanes.
Mid-latitude cyclones can have winds as strong as what is associated with
a weak hurricane and can cover very large areas upto 1000km in width
lasting several days within the same region.

Tropical Cyclones
The term tropical cyclone is used to describe hurricane-like storms that
originated over tropical oceans with Surface atmospheric pressure in the
center extremely low. This happens when the I.TC.Z is farthest from the
equator during the month s of august and September in the southern
hemisphere and in February and March in the northern hemisphere. They
are also smaller than the depressions, measuring on average 550km in
diameter. The eye of the cyclone may be 20 to 50km in diameter.

Page | 43
Prefer Calling Sir Obiero Amos @ 0706 851 439 for F1-F4 All Subjects Notes
Tropical cyclones have no true fronts associated with them like the
depressions of the polar front. This is because the air masses involved (SE
and NE trade winds) have similar characteristics.
Hurricanes are intense cyclonic storms that develop over the warm oceans
of the tropics. To be classified as a hurricane, sustained wind speeds must be
greater than 118 km/h reaching 240km/h at the storm's center. Wind speed
in a hurricane is directly related to the surface pressure of the storm. The
part of the cyclone in which wind moves in a circular manner and at the
same time rises is called a vortex.
The strongest winds and heaviest precipitation are found in the area next to
the eye where a vertical wall of thunderstorm clouds develops from the
Earth's surface to the top of the troposphere.
Before the arrival of a cyclone, the air becomes very still, temperature and
humidity are high, then gusty winds develop and thick clouds appear in the
sky. The arrival of the front of the vortex brings about very violent winds
blowing from one direction at speeds of upto 240km/h at the eye. Dense
clouds and torrential rains follow and visibility reduced. When the eye
arrives, calm conditions and a clear sky resume though short-lived. This is
followed by violent winds blowing from the opposite direction.
As long as the vortex lasts, heavy rains and dense clouds are experienced.
When the vortex passes, calm conditions resume. Hurricanes are seldom
motionless after their initial formation. On average, hurricanes that form in
the North Atlantic and North Pacific move in a west or northwestward path.
In reality, the track taken by any individual storm is often very chaotic.
Hurricanes can suddenly change both their speed and direction of travel
depending on friction and available obstacles. Cyclones also cause a lot of
damage.

Tornadoes
A tornado is a vortex of rapidly moving air associated with some severe
thunderstorms.
They develop over land and are mostly developed in south east USA where
they are called twisters. Tornadoes that travel across lakes or oceans are
called waterspouts. Winds within the tornado funnel may exceed 500

Page | 44
Prefer Calling Sir Obiero Amos @ 0706 851 439 for F1-F4 All Subjects Notes
kilometers per hour. High velocity winds cause most of the damage
associated with these weather events. Tornadoes also cause damage
through air pressure reductions. The air pressure at the tornado center is so
low (approximately 800mb) that many human made structures collapse
outward when subject to pressure drops of this magnitude.
Tornado is accompanied by heavy rain and thunder because of the rapidly
rising moist air in it.

Anticyclone
Also high pressure systems.
An anticyclone is a region of high atmospheric pressure relative to the
surrounding air, generally thousands of kilometres in diameter. The name
anticyclone is because the Anticyclonic circulation has a local circulation that
is opposed to the Earth's rotation. Winds, generally light, circulate around
the high pressure centre in a clockwise direction in the Northern
Hemisphere and anticlockwise in the Southern Hemisphere. This is a
consequence of the Coriolis force.

High pressure systems in the Northern hemisphere.


Anticyclones are typically relatively slow moving features. Subsiding air
compresses as it descends, causing adiabatic warming. The eventually
warmer and drier air suppresses cloud formation and thus anticyclones are
usually associated with fine weather in the summer and dry, cold, and
sometimes foggy weather in the winter.

Page | 45
Prefer Calling Sir Obiero Amos @ 0706 851 439 for F1-F4 All Subjects Notes
G. WIND.
This refers to air in motion. Air is composed of various gases constantly
moving in various directions. Surface wind is the wind blowing near the
Earth's surface. Wind is a medium of transfer of heat and moisture and is
responsible for the movement of clouds.
Windspeed is significant in explaining weather conditions of a place as it
determines wind strength. Strong winds lead to little rain. Wind direction
helps to explain the characteristics of the particular wind in terms of
temperature and moisture content.
Winds are named according to the direction from which they are blowing
and according to the temperature and moisture characteristics.

The planetary winds.


These are the major winds blowing over the earth consistently caused and
controlled by the major pressure belts of the earth. They are also known as
prevailing winds (winds that blow in a place most frequently)

Planetary winds
They include;
a) The North East Trade Winds. They blow from the sub-tropical zone to the
equatorial low pressure belt.
b) The westerlies. They blow from the sub-tropical high pressure belt to the
temperate low pressure belt. They start as the south westerlies but are
adjusted to become westerlies.

Page | 46
Prefer Calling Sir Obiero Amos @ 0706 851 439 for F1-F4 All Subjects Notes
c) Polar Easterlies. They blow from the polar high pressure region to the
temperate LP zone at latitude 60
N.
d) The south East Trade winds. These winds blow from the sub-tropical HP
belt towards the equatorial low pressure zone.
e) The westerlies. They blow from the sub-tropical HP belt to the temperate
LP zone of the S.H.
f) The Polar Easterlies. They originate from the polar HP region and blow
towards the temperate low pressure region

NB; winds shift according to the shifting of pressure belts that shift due to
seasonal changes in temperature affecting the atmosphere.

Monsoon Winds
Monsoons are regional scale wind systems that predictably change direction
with the passing of the seasons. These wind systems are created by the
temperature contrasts that exist between the surfaces of land and ocean.
However, monsoons are different from land/sea breezes both spatially and
temporally.
Monsoons occur over great distances, and their pattern of wind flow is on
annual time scale.
During the summer, monsoon winds blow from the cooler ocean surfaces
onto the warmer continents. In the summer, the continents become much
warmer than the oceans. Precipitation is normally associated with the
summer monsoons. This precipitation can be greatly intensified by

Orographic uplift.
In the winter, the wind patterns reverse as the ocean surfaces are now
warmer. The winter monsoons bring clear dry weather and winds that flow
from land to sea.
The Asiatic monsoon is the result of a complex climatic interaction between
the distribution of land and water, topography, and tropical and mid-
latitudinal circulation.

Page | 47
Prefer Calling Sir Obiero Amos @ 0706 851 439 for F1-F4 All Subjects Notes
In the summer, a low pressure center forms over northern India and
northern Southeast Asia because of higher levels of received solar insolation.
Warm moist air is drawn into the thermal lows from air masses over the
Indian Ocean. Summer heating enhances rainfall in Southeast Asia, in the
Arabian Sea, and in South India.
When autumn returns to Asia easterly jet stream is replaced with strong
westerly winds in the upper atmosphere.
Subsidence from an upper atmosphere creates a surface high pressure
system that dominates the weather in India and Southeast Asia.

Local winds.
These are winds that occur either regularly or for a short period of time and
affected a limited area and for a short period of time.
They have distinct characteristics and produce significant effects on weather
of the areas over which they blow. Examples.

a) Sea and Land Breezes


This is the wind which occurs in response to differences in temperature
between a body of water and neighboring land. During the daytime land
heats up much faster than water as it receives solar radiation from the Sun.
The warmer air over the land then begins to expand and rise forming a
thermal low. At the same time, the air over the ocean becomes a cool high
because of water's slower rate of heating.

Rising warm Descending

air cold air

Se a Breeze

Land

Ocean

Page | 48
Prefer Calling Sir Obiero Amos @ 0706 851 439 for F1-F4 All Subjects Notes
Air begins to flow as soon as there is a significant difference in air temperature and

pressure across the land to sea gradient. The heavier cooler air over the ocean to move

toward the land and to replace the air rising in the thermal low. This localized air flow

system is called a sea breeze. Sea breeze usually begins in midmorning and reaches its

maximum strength in the later afternoon when the greatest temperature and pressure

contrasts exist. It dies down at sunset when air temperature and pressure once again

become similar across the two surfaces.

Daytime development of sea breeze.


At sunset, the land surface stops receiving radiation from the Sun and
begins losing heat energy at a much faster rate than the water surface.

Cool air

sinks
Rising air helps

form clouds

Sinking air spreads

along surface

Land Breeze

Land cools down


Ocean water is warmer

The land surface being cooler than the water becomes a thermal high
pressure area. The ocean becomes a warm thermal low. Wind flow now
moves from the land to the open ocean. This type of localized air flow is
called a land breeze.

Page | 49
Prefer Calling Sir Obiero Amos @ 0706 851 439 for F1-F4 All Subjects Notes
Night development of land breeze
b) Mountain and Valley Breezes
Mountain breezes are part of local wind systems which are common in
regions with great topographic relief. They are also known as katabatic and
anabatic winds respectively.
Katabatic wind (from the Greek: katabaino - to go down) is the generic term
for downslope winds flowing from high elevations of mountains, plateaus,
and hills down their slopes to the valleys or planes below. Katabatic winds
result out of air in contact with upper level ground being cooled by
radiation, increasing in density, and then flow downhill and along the valley
bottom.
For example radiation cooling during nighttime can cause a katabatic flow
in the early morning.

Mountain Breeze

A valley breeze develops during the day as the Sun heats the land surface
and air at the valley bottom and sides. As the air heats it becomes less dense
and buoyant and begins to flow gently up the valley sides. Vertical ascent
of the air rising along the sides of the mountain is usually limited by the
presence of a temperature inversion layer.
If conditions are right, the rising air can condense and form into cumulus
clouds.

Page | 50
Prefer Calling Sir Obiero Amos @ 0706 851 439 for F1-F4 All Subjects Notes
Valley breeze

c) The Harmattan winds.


These are N.E winds which blow from the Sahara, across West Africa
between November and March. They are dusty and dry winds since they
originate and blow over the desert areas. The winds are a product of
dissimilar pressure between North Africa (HP cell) and Southern Africa (LP
cell).
The Harmattan winds blow towards the gulf of Guinea but do not reach the
sea because the pressure is lower over the Gulf Coast and the South East
winds from the HP cell over the south of Atlantic keep them at bay.

d) The Leveche, Sirocco, chili and Khamsin.


These are local winds which blow from the high pressure cell over the Sahara
during spring bowing northwards. The Khamsin is a hot, dry and dusty
south or south-east wind occurring in N. Africa, around the E Mediterranean
and the Arabian Peninsula between April and June. The term is also applied
to very strong southerly or south-westerly winds over the Red Sea.
Like the sirocco, the khamsin is usually blowing ahead of depressions which
move eastward or northeastward in the Mediterranean Sea or across N.
Africa, with high pressure to the east. The name is derived from the Arabic,
khamsun or hamsin, meaning fifty, for the approximate period of days during
Page | 51
Prefer Calling Sir Obiero Amos @ 0706 851 439 for F1-F4 All Subjects Notes
which it blows. In other parts of N Africa and the Mediterranean similar
winds are variously known as sirocco, ghibli and leveche.

e) Föhn (foehn) wind


The Föhn is a warm, dry, gusty wind which occurs over the lower slopes on
the lee side of a mountain barrier. It is a result of forcing stable air over a
mountain barrier. Föhn winds occur quite often in the Alps (where the name
föhn originated) and in the Rockies (where the name Chinook is used). f)

Föhn (foehn) wind

Berg winds.
They originate from the plateau of South Africa during winter. At that time,
a HP cell lies over the plateau and it is a region of descending air. The winds
blow out towards the S.E, South and S.W as berg winds. They are warm
winds since on descending they are compressed by the increasing
atmospheric pressure.

g) Mistral.
The cold winds that blow from France southwards towards
the Mediterranean Sea. h) Bora
These are Winds that blow from the Dinaric Alps towards the Adriatic Sea.

Factors influencing wind direction and speed.


1. Pressure gradient. Air flows from a region of HP to a region of LP. Steep
pressure gradient (usually determined by the distance between two
regions of different pressure) results in strong winds.

Page | 52
Prefer Calling Sir Obiero Amos @ 0706 851 439 for F1-F4 All Subjects Notes
2. Coriolis Effect. The Coriolis Effect is a force which acts upon any moving
body or a parcel of air in an independently rotating system, such as the
Earth.

An object or current moving above the


Earth in a generally northerly or
southerly direction (away from the
equator) will have a greater eastward
velocity than the ground underneath, and
so will appear to be deflected in relation
to the rotation of the Earth. This
deflection acts towards the right (or
clockwise) in the, in the Northern
Hemisphere and towards the left (or anti-
clockwise) in the Southern Hemisphere.
Winds blowing closer to the poles will be deflected more than winds at the same
speed closer to the equator. The Coriolis force is zero right at the equator and
becomes a

3. Centrifugal force. When air streams move along a curved isobar in


anticyclones, there is a centrifugal force acting outwards from the centre
of the curvature. Thus in the northern hemisphere, air will tend to move
clockwise in anticyclones and anticlockwise in cyclones.
4. Effect of friction. Topographical obstacles create friction on moving air
causing it to lose strength and even change direction.

MEASUREMENT AND RECORDING OF WETHER ELEMENTS.


As a variety of atmospheric conditions need to be recorded, a wide range of
equipment is needed to obtain that information.
Details of some of this specialist meteorological equipment are given below.

Page | 53
Prefer Calling Sir Obiero Amos @ 0706 851 439 for F1-F4 All Subjects Notes
1. TEMPERATURE.
The temperature of the atmosphere is measured using an instrument called
a thermometer.
Thermometer
Thermometers measure temperature. "Thermo" means heat and "meter"
means to measure.
Thermometer measures temperature in degrees centigrade (°C) or degrees
Fahrenheit (°F) using a liquid such as mercury that expands when it warms
up.
It then moves up a thin tube marked with a temperature scale, and will fall
back down the tube as the temperature falls and the liquid contracts.

Comparison of the Centigrade and Fahrenheit scales.

There are many types and many uses for thermometers, as detailed below.
• Maximum thermometer.
• Minimum thermometer.
• Six’s thermometer (combined maximum and minimum
thermometer)

Page | 54
Prefer Calling Sir Obiero Amos @ 0706 851 439 for F1-F4 All Subjects Notes
• Maximum thermometer.
This thermometer uses mercury to record the highest
temperature reached in a day. When the temperature rises, the
mercury expands and pushes the index forward.

When the temperature falls, the mercury contracts


leaving the index behind. The maximum
temperature reached in the previous 24hours is
shown by the index that was in contact with the
mercury last. In the illustration, the max
temperature was about 28.3°C

Marker showing the maximum temperature. (Maximum of about 28.3°C)


After taking the readings, the thermometer is set by bringing the metallic
index into contact with mercury using a magnet.

Minimum thermometer.
It records the lowest reached in a day using alcohol. When the temperature
falls, the alcohol contracts and its meniscus pulls the index towards the bulb.
When the temperature rises, the alcohol expands leaving behind the index.
The minimum temperature in the previous 24 hours is obtained by reading
the scale at the end of the index nearest to the meniscus.
To reset the thermometer, raise the bulb of the
thermometer gently.

Six’s or Maximum and minimum thermometers


Six's thermometer, invented by Englishman James Six in 1782,is a registering
thermometer which can record the maximum and minimum temperatures
reached over a period of time, for example 24 hours.
It is used to record the extremes of temperature at a location.

Page | 55
Prefer Calling Sir Obiero Amos @ 0706 851 439 for F1-F4 All Subjects Notes
Maximum minimum thermometer-diagram.
Detail of the thermometer bulbs of the maximum
minimum thermometer. The lefthand (minimum arm)
bulb is full of alcohol. This bulb measures the
temperature by the expansion and contraction of the
liquid. The right-hand (maximum arm) bulb contains
alcohol and a bubble of low-pressure gas or alcohol
vapour. This bulb accommodates the expansion in the
other bulb and allows the train of alcohol and
mercury to move in the U-shaped tube as the temperature changes. In the bend of
the U is a section of mercury, which is pushed around the tube by the thermal
expansion and contraction of the alcohol in the first bulb as it responds to the
external temperature. The near vacuum in the other bulb allows free movement of
the alcohol and mercury. It is the alcohol which measures the temperature; the
mercury indicates the temperature reading on both scales.

The maximum and minimum readings are recorded by two small steel
markers which are sprung into the capillary tube so that they can slide by
being pushed by the mercury.
If the temperature rises, the maximum scale marker will be pushed. If it falls,
the moving mercury will push the minimum scale marker.
As the temperature varies, the markers will remain in their positions unless
the temperature becomes higher (for maximum) or lower (for minimum)
than already recorded, in which case the relevant marker is pushed further.
Typically the thermometer is reset every 24 hours to measure the diurnal
temperature variation.

Page | 56
Prefer Calling Sir Obiero Amos @ 0706 851 439 for F1-F4 All Subjects Notes
Usefulness of the records of the minimum and maximum temperatures.
They are used in the following calculations.
a) Mean diurnal temperatures (Diurnal means temperature). This is obtained
by finding the average of the maximum and minimum temperatures
recorded during the day (24hrs).
Daily max temp. + Daily min temp.
2
E.g Maximum=40 C, Minimum =22 C
Mean Daily Temp. = 40 C +22 C

2 =31 C
b) Diurnal Range of temperature/Daily temperature Range. This is the
difference between the maximum and minimum temperature
recorded in a day (24hrs)
Diurnal Range of temp=Daily max temp. - Daily min temp
E.g Maximum=40 C, Minimum =34 C
Diurnal range of Temp. = 40 C - 34 C =6 C

c) Mean monthly temperature. The average temperature of the month is


obtained by adding all the means in a month and dividing by the
number of days in that month.
d) Mean Annual Temperature. This is the average temperature for the year.
It is obtained by adding all the monthly mans and dividing by 12 (the
number of months in a year.
e) Annual Range of temperature. This is the difference between the highest
and the lowest mean monthly temperatures in a given year.

2. RAINFALL
Rain gauge is the instrument used to measure the amount of liquid
precipitation over a set period of time.
Rain gauges essentially comprise carefully calibrated collecting buckets.
The unit of rainfall that is used is equal to the depth of water which would
have resulted on the ground if none of the water which fell could escape

Page | 57
Prefer Calling Sir Obiero Amos @ 0706 851 439 for F1-F4 All Subjects Notes
(either by seepage into the ground, or by evaporation) and if the water
were spread uniformly over the whole ground.
Thus a rainfall of "15 mm" means that if all of the rain were spread uniformly
across the landscape, it would have had a depth of 15 mm.

The gauge is sunk into the ground such


that the rim of the funnel is 30 cm
above the ground level to prevent
splashes of water or surface runoff
from getting into the gauge when it
rains. After it rains, the readings are
taken and recorded

Location of a rain gauge.


a) An open space to avoid trees, buildings or any other object that can
direct water into it.
b) The cylinder should be maintained at a height of 30cm above the
ground to avoid water splashing into the funnel.
c) The outer case should be sunk into the ground to prevent evaporation
of the rainwater collected in the jar.
NB; the readings are usually taken and recorded once in 24hrs after which
the cylinder is emptied ready for the next rain.

From the rain data collected, the following calculations are made.
a) The monthly rainfall totals. The sum of the rainfall recorded during the
month.
b) The annual rainfall total. The sum of the rainfall received in the year.

Page | 58
Prefer Calling Sir Obiero Amos @ 0706 851 439 for F1-F4 All Subjects Notes
c) Mean monthly rainfall. The sum of the monthly rainfall for a particular
month observed over several years e.g 20 or 30 years and divided by the
number of observation years.
d) The mean annual rainfall. The sum of the mean monthly rainfall for 12
months of the year.
NB; Rainfall mean is not calculated in the same way as temperature. It is not
the exact arithmetic average.
Rainfall is not naturally distributed evenly in all the months of the year.

3. SUNSHINE.
The duration of sunshine for the day is measured using the Campbell stokes
sunshine recorder.
It is a glass sphere which focuses the sun’s rays on a sensitized paper
mounted ob a metal frame.
Since the rays are focused on the sensitized paper, they burn a line as the
sun moves across the sky. There are calibrations in hours on the margin of
the paper. Continuous sunshine will produce continuous burn whereas
intermittent sunshine produces unburnt parts on the paper.
The length of all the burnt sections is added so as to obtain the total hours of
sunshine for the day. The sensitized paper is changed every day. A measure
of the duration of sunshine for a week or a month is calculated using this
method.
4. HUMIDITY.
Humidity is measured using a wet and dry bulb
Wet and dry bulb thermometer thermometer also known as Thermo-Hygrometer or
Psychrometer.
Two mercury-filled thermometers are mounted together on a
bracket, with one extending a few inches below the other.
The bulb of one of the thermometers is covered with a muslin
wick, which is moistened with distilled water. This wet-bulb
thermometer is cooled by evaporation to a value below the
temperature shown by the dry-bulb thermometer.

Page | 59
Prefer Calling Sir Obiero Amos @ 0706 851 439 for F1-F4 All Subjects Notes
The computation of the humidity is carried out by comparing the two readings of the
thermometers, since the difference between them depends on humidity.
The difference between the two thermometer readings is used to calculate relative
humidity using a humidity table.

NB.
a) If the atmosphere is saturated, there is no evaporation and so the
atmosphere reading on both thermometers are the same. Relative
Humidity is 100%.
b) When the difference is small, it means that relative humidity is very high.
c) When the difference is large, it means that relative humidity is very low.

Humidity is also measured using a Hygrometer.


This instrument uses the fact that human hair changes in length in response
to changes in humidity, with the hair becoming longer when the relative
humidity increases.
This minute contractions and expansions of the hair strands are magnified
by a system of levers and transferred to an inked pen which traces a record
on a chart wrapped around a rotating drum.
Thus as the humidity changes, so the needle adjusts in position, in response
to changes in the length of the hair.

Hair tensions dial hygrometer with a nonlinear scale


Page | 60
Prefer Calling Sir Obiero Amos @ 0706 851 439 for F1-F4 All Subjects Notes
5. WIND
Measurements of the wind require two parameters - speed and direction.
It is measured by an anemometer (speed) or wind vane (wind direction) at
a standard height of 10 m above ground in an area where the distance
between the instrument and any obstruction is at least 10 times the height of
the obstruction. This distance is known as fetch and is kept in order to avoid
the influence of eddies. Wind vane.

The wind vane shows thedirection. It pivots on a


vertical shaft. When the wind blows the arm swings
until the pointer faces the wind.

Anemometer
An anemometer, also called a wind gauge. Anemometers can measure wind
speed, wind direction, and other information.
The cup anemometers are made of cups that catch the wind and make the
central shaft spin.
Three cups are mounted at equal intervals on a central shaft so that it can turn
freely.
The shaft has a magnet attached to it which generates a small electrical current as
it spins inside coils of copper.

The anemometer allows the wind


speed to be measured. It generally
consists of three of four cups, which
rotate freely; the faster the wind, the
faster the cups rotate.

Page | 61
Prefer Calling Sir Obiero Amos @ 0706 851 439 for F1-F4 All Subjects Notes
The number of times it rotates every minute gives the wind speed. The
number of rotations is recorded by a small counter at the base of the shaft.

This anemometer measures the wind speed, and the wind


vane on the left side of the image measures the direction
the wind is blowing.

Wind Sock – shows the speed and direction of the wind. They are most often
used at airports, seaports and on other open areas such as mountain roads.
Wind Vane – measures wind direction. It is always recorded as the direction
from which the winds are blowing, i.e.: a south-westerly wind is blowing
from the south-west.

6. MEASURING PRESSURE.
Atmospheric pressure can be measured with a mercury barometer and an
aneroid Barometer.

Mercury Barometer
A mercury Barometer indicates the height of a column of mercury that
exactly balances the weight of the column of atmosphere over the
barometer.The atmospheric pressure provides the force necessary to push
the mercury up the evacuated tube.

Page | 62
Prefer Calling Sir Obiero Amos @ 0706 851 439 for F1-F4 All Subjects Notes
Atmospheric pressure is also measured using an aneroid barometer.

Aneroid barometers
An aneroid barometer can be used in place of a mercury barometer. This
instrument relies on a sealed, flexible unit which contains air at a fixed
volume.

Aneroid barometer consists of a


collapsible metal box from which air
has been removed. The metal box
responds to changes in pressure by
expanding when pressure falls or by
collapsing slightly when pressure
increases. The movement is
conveyed by a series of levers to a
point which moves a graduated
scale. Unlike the mercury
barometer, the aneroid barometer
has the advantage of recording
changes in atmospheric pressure
as they occur during the day on the
rotating drum

Page | 63
Prefer Calling Sir Obiero Amos @ 0706 851 439 for F1-F4 All Subjects Notes
7. EVAPORATION.
The instrument used to measure the rate and amount of evaporation is
known as an evaporimeter. There are two kinds;
a) Piche Evaporimeter.
It consists of a glass tube filled with water. The open side is slightly fixed to
the tube by means of a steel wire spring.
The tube is inverted and suspended by means of a string. As the blotting
paper absorbs water from the glass tube, evaporation starts to take place
particularly when there is a lot of sunshine.
The rate and amount of evaporation is determined by looking at the scale on
the outside of the glass tube. b) Tank Evaporimeter.
It is a fairly large and shallow tank in which water is exposed in an open air.
The falling level of the water can be measured at intervals.

Pan/ tank evaporimeter.

8. MEASUREMENT OF CLOUD COVER.


This is estimated by determining what fraction or %age of the sky is covered.
The remote sensing method is more accurate in this measurement.
A computerized camera on a spaceship revolving around the earth is able to
measure the amount of cloud cover around the earth.
The cloud cover of a place determines the amount of sunshine that filters
through to the surface of the earth.
Visibility can be measured using a standard scale consisting of a set of letters
each of which represents a specific distance.
Page | 64
Prefer Calling Sir Obiero Amos @ 0706 851 439 for F1-F4 All Subjects Notes
THE ATMOSPHERE
It is the layer of gases surrounding the earth which are held by the earth’s
gravitational pull.
It rotates with the earth and is about 330km thick.
Atmospheric air refers to the mechanical mixture of gases in the atmosphere.
Atmospheric Composition
Of the gases in the atmosphere, nitrogen, oxygen, water vapor, carbon
dioxide, methane, nitrous oxide, and ozone are extremely important to the
health of the Earth's biosphere.
The gases are colourless and odourless.

Atmospheric composition.
Gas % by Importance for weather and climate
volume
Nitrogen 78.09 Mainly passive
Oxygen 20.95 Mainly passive
Source of clouds and precipitation, reflects and
Water absorbs incoming radiation, keeps global
0.2 - 0.4
vapour temperatures constant; provides majority of
natural 'greenhouse effect';
Absorbs long wave radiation from earth and so
Carbon
0.03 contributes to GHE, its increase due to human
dioxide
activity is a major cause of global warming
Ozone 0.00006 Absorbs incoming UV
Dust trace Absorbs/reflects incoming radiation; forms
condensation nuclei
Sulphur
dioxide,
nitrous trace Affects radiation: causes acid rain
oxide,
methane

Page | 65
Prefer Calling Sir Obiero Amos @ 0706 851 439 for F1-F4 All Subjects Notes
Nitrogen and oxygen are the main components of the atmosphere by
volume (approximately 99% of the dry atmosphere).
Both of these gases have very important associations with life and are,
together with carbon dioxide, abundant in the first 90 km of the earth’s
surface (99.9%).
Water vapor and dust particles vary in concentration in the atmosphere both
spatially and temporally. Others are helium, neon and hydrogen.

The Layered Atmosphere


The Earth's atmosphere contains several different layers that can be defined
according to air temperature as displayed below.
Vertical change in average global atmospheric temperature.

Page | 66
Prefer Calling Sir Obiero Amos @ 0706 851 439 for F1-F4 All Subjects Notes
The atmosphere contains four different layers

1. The troposphere.
It is the lowest layer. It depth varies from about 8 to 16 kilometers. Greatest
depths occur at the tropics where warm temperatures cause vertical
expansion of the lower atmosphere.
About 80% of the total mass of the atmosphere is contained in troposphere
(75% of the gases in the atmosphere and 90 % of the water vapour).
It is also the layer where the majority of our weather occurs. It is a life
supporting layer since it has oxygen, nitrogen and CO2. Pressure falls with
increase in height. The speed of wind increases with height. With increasing
height, air temperature drops uniformly with altitude at a rate of
approximately 6.5° Celsius per 1000 meters. This phenomenon is commonly
called the Environmental Lapse Rate.
At the upper edge of the troposphere is a narrow transition zone known as
the tropopause.

2. The stratosphere.
This layer extends from an average altitude of 11 to 50 kilometers above the
Earth's surface. This stratosphere contains about 19.9% of the total mass
found in the atmosphere. Very little weather occurs in the stratosphere.
Occasionally, the top portions of thunderstorms breach this layer.
In the first 9 kilometers of the stratosphere, temperature remains constant
with height. A zone with constant temperature in the atmosphere is called
an isothermal layer. From an altitude of 20 to 50 kilometers, temperature
increases with an increase in altitude (negative lapse rate).
The higher temperatures found in this region of the stratosphere occurs
because of a localized concentration of ozone gas molecules that absorb
ultraviolet sunlight creating heat energy that warms the stratosphere. Ozone
is primarily found in the atmosphere at varying concentrations between the
altitudes of 10 to 50 kilometers. This layer of ozone is also called the ozone
layer . The ozone layer is important to organisms at the Earth's surface as it
protects them from the harmful effects of the Sun's ultraviolet radiation.

Page | 67
Prefer Calling Sir Obiero Amos @ 0706 851 439 for F1-F4 All Subjects Notes
Winds are light in the lower parts but increase with height. Pressure
continues to fall as air is dry.
The layer is a positive shield against meteorites that usually burn out as they
enter the earth’s gravitational pull

3. The mesosphere.
Separating the mesosphere from the stratosphere is transition zone called
the stratopause. It extends from 50-80km
Temperature decreases with increase in height (positive lapse rate). In the
mesosphere, the atmosphere reaches its coldest temperatures (about -90°
Celsius) at a height of approximately 80 kilometers. The layer experiences
the strongest winds with 3000km/h speed. At the top of the mesosphere is
another transition zone known as the mesopause

4. The thermosphere.
The last atmospheric layer has an altitude greater than 80 kilometers.
Temperatures in this layer can be greater than 1200° C. These high
temperatures are generated from the absorption of intense solar radiation by
oxygen molecules (O2). The air in the thermosphere is extremely thin with
individual gas molecules being separated from each other by large distances.

WEATHER FORECASTING.
It is the prediction of the state of the atmosphere for a given place over a
period of 24 to 48 hours. Prediction of weather can be done using a past
weather record or prevailing weather conditions.
Methods of weather forecasting.
1) Ancient methods. In ancient times it was believed that weather was
governed by gods. Weather forecasting in the 17th century was based on
astrological, local, traditional and religious theories. Some of the
traditional weather forecasts showed careful observation expressed in
form of poems and jingles that were passed on from generation to
generation.

Page | 68
Prefer Calling Sir Obiero Amos @ 0706 851 439 for F1-F4 All Subjects Notes
2) Weather lore. This refers to a body of traditional facts and beliefs relating
to weather like a halo around the moon, a rainbow, croaking of frogs,
butterfly flights, migration of birds, changes in wind direction, clouds,
and position of the moon and the intensity of the sun’s radiation.
However it is not strictly reliable.
3) Modern methods. They are more reliable and accurate. World
Meteorological Organization has set up a conventional system of
weather recording and interpretation for international use. In Kenya,
weather forecasting is based on records from meteorological stations
and remote sensing centre. Dagoretti id the HQ of meteorological
stations in East and Southern Africa.

Weather forecasting is
hindered by; ~
Inadequate data.
~ Inaccurate/unreliable data brought about by faulty equipment.
~ Intervening factors like slope of the land, nature of the vegetation, soil
moisture conditions and winds lead to variation in temperature at
different areas adjacent to each other. ~ Inadequate personnel in
developing countries
~ Vagaries of nature like storms and earthquakes.

Significance of weather forecasting.


a. It helps in determining the farmer’s calendar to enable the farmer adjust
his farming activities.
b. It helps people to determine the suitable clothing, suitable housing,
fishing habitats, times for air and sea travels and sporting activities.
c. It helps in guiding tourists activities such as visiting game parks.

d. Military personnel may also benefit as they can plan their military
activities based on the expected weather conditions.
e. It helps to know beforehand about the possible natural calamities and the
related weather in order to take precautions.

Page | 69
Prefer Calling Sir Obiero Amos @ 0706 851 439 for F1-F4 All Subjects Notes
STATISTICS.
It is the exact facts and figures collected and arranged in a systematic
manner.
It is an art or science which is concerned with the interpretation of numerical
information.
STATISTICAL DATA.
This is the actual facts and figures collected from various areas and arranged
in an organized manner.
STATISTICAL METHODS.
These are the techniques of collecting, recording, analyzing, presenting and
interpreting statistical data with a view to drawing valid and useful
conclusions from them.
Importance of statistics.
a) Illustration of relationship between two varying quantities. For example the
effects of an increase in cultivated land acreage on the production of
beans.
b) Summary of geographical information. A lot of facts can be summarized in
a smaller space.
c) Explanation of geographical phenomena. For example, climatic data can be
used to explain the dominance of a certain vegetation type.
d) For comparison purposes. We can use statistical data to compare
production of a particular commodity from different areas.
e) It is used to make prediction. Climatic data can be used to predict future
trends in weather or climatic patterns. E.g prediction of an impending
drought by using trends in temperature and rainfall.
f) It shows changes through time. For example imports of oil over the past 10
years.
g) For planning purposes. Census figures helps in planning for provision of
social amenities.
h) It gives precise rather than generalized information especially about
rainfall.

Page | 70
Prefer Calling Sir Obiero Amos @ 0706 851 439 for F1-F4 All Subjects Notes
TYPES AND SORCES OF STATISTICAL DATA.
There are two main categories; Primary data and secondary data.
1. Primary Data. These are facts and figures collected from the field as first
hand or original information. For example the mean daily temperature
you personally recorded from a school weather station. Such data is
collected using observation, interviewing, counting, and measurement
methods.
2. Secondary data. This is derived data which is available in stored sources
like text books, reference books, magazines, maps and video/audio
tapes.

NATURE OF STATISTICAL DATA.


There are two categories;
a) Discrete data; also known as non-continuous data, it is given in whole
numbers. For example 16 elephants, 35 boys. It cannot be given in
fraction.
b) Continuous Data. It is continuously distributed over time. For example
distance, population growth rates, change in trade, temperature. It can
be given in fractions and decimals. For example 24.5 grams, 2.5 km,
26%, 22.5%.

METHODS OF DATA COLLECTION DATA.


1. Use of questionnaire.
A questionnaire is a list of questions related to the topic of research to be
answered.
Characterics of good questionnaire questions.
a) The questions should be simple and clear. Avoid ambiguous questions.

b) The questions should not be too many to discourage the respondent.

c) The questions should be arranged in a logical order from the simplest to


the difficult one.
d) Nature of questions. Avoid annoying questions especially those touching
on the respondent’s private life.
e) Objectivity. They should be free from bias. Avoid leading questions.

Page | 71
Prefer Calling Sir Obiero Amos @ 0706 851 439 for F1-F4 All Subjects Notes
Types of questionnaires
a) Personal interview questionnaires. This has a series of guide questions
to guide the respondent in giving answers. The response is stored in
written form in space provided or on tape.
b) Rigid Questionnaire. Space is provided for each question. The
respondent sometimes responds in the absence of the respondent.

Types of questions in a questionnaire.


a) Open-ended questions. The respondent answers as she/he feels. For
example. What products do you sell?
b) Closed/rigid questions. This are structured or multiple choice questions
that limit the respo\ndent to selections without deviation.

Advantages of using a questionnaire.


a) Comparison can be made easily since similar questions are used for all
respondents.
b) The data obtained is first hand.

c) The physical appearance of the conductor of the field study is not


necessary since questions are straight forward.
d) A lot of information can be collected within a questionnaire if the
questions are well prepared.
e) It saves time since all respondents are handled at the same time.

f) Rigid questions when posted reduce fieldwork expenses.

g) It gives an opportunity to gauge the accuracy of the answers (through


direct contact with the respondent).

Disadvantages.
a) Some respondents may write down inaccurate answers thus rendering
the data inaccurate.
b) Data analysis may become difficult in a case where different answers
are obtained for the same question.
c) Some questionnaires may be returned unfilled or not returned at all in
case of a posted questionnaire. This may be in case of lazy or
uncooperative respondents.
Page | 72
Prefer Calling Sir Obiero Amos @ 0706 851 439 for F1-F4 All Subjects Notes
d) Where there is a language barrier or in case of illiteracy, the technique
would not be useful.

2. Interviewing.
Interviewing is when questions are asked to respondents and answers
received from them to obtain information about desired geographical aspect.
An interview may be conducted face to face or on telephone.
Below are some of the guidelines on how to achieve a conducive
atmosphere for an interview; a) Approach people politely.
b) Create a warm and friendly atmosphere for respondents.
c) Assure the respondents that the information they give remains
confidential.
d) Avoid unnecessary interruptions during the interview.
e) Do not suggest answers. Let all the answers come independently
from the respondents. Advantages of the interview technique.
a) It gives first-hand information through direct responses from the
resource persons.
b) The interviewer can seek clarification from the respondent in case of
ambiguity in answers.
c) The interviewer is free to seek for more information by initiating
further discussions or by asking other questions.
d) Information can also be obtained from people who cannot read or
write.
e) The interviewer has the opportunity to create a good rapport with the
respondent and this would ensure that reliable answers are given.
f) The interviewer can also gauge the accuracy of the responses.

Disadvantages.
a) It is time consuming since the interviewer has to handle one person at
a time.
b) It is expensive and tiresome since the interviewer has to move to
different places to meet the respondents.

Page | 73
Prefer Calling Sir Obiero Amos @ 0706 851 439 for F1-F4 All Subjects Notes
c) It can easily be prone to language barrier where the interviewer and the
respondent do not speak the same language. Use of interpreters might
distort some information.
d) Where resource persons may have forgotten, information on events in
the past may be unreliable.
e) The respondent may sometimes through exaggeration give wrong
information or even deliberately mislead.

3. Content analysis
This is a technique employed to collect data from secondary sources like text
books, magazines, newspapers, census reports, statistical abstracts, atlases
etc.
The techniques employed in content analysis include;
a) Reading and extracting relevant information.

b) Watching films.

c) Viewing photographs.

d) Listening to audio tapes using tape recorders. Advantages of content


analysis.
a) It is easy to get data especially if it is already analyzed.

b) It is cheap because it does not involve extensive traveling.

c) It saves time since all the required information may be obtained in one
place like the library.
d) It is possible to get old data because it is already stored in various
sources.
Disadvantages.
a) It may be difficult to verify the accuracy of available data.

b) The data may be irrelevant to current trends because it is out of date.

c) Up-to-date information may not be readily available.

Page | 74
Prefer Calling Sir Obiero Amos @ 0706 851 439 for F1-F4 All Subjects Notes
4. Sampling
A sample may be regarded as a specimen or a small portion of a whole or a
bigger population, or part of the area under study taken to show what the
rest is like.
For example, it could be a percentage of total population to be interviewed
or soil sampled rock type etc. Sampling method is useful when a
questionnaire is used and when it is impossible to get the views of
everybody.
Where the area under study is too large, the selection of a representative
sample (10% of the total population) is necessary. Types of sampling
techniques.
a) Random sampling. It is the technique that involves choice of places to be
used as bases for study randomly. For example through dividing the
study area into imaginary squares using lines then picking the study area
through some kind of raffle.
b) Systematic sampling. This method is applied where distribution of
phenomenon is even. E.g. a forest of cypress trees.
c) Stratified sampling. In this method, the study samples are selected
according to their groups or classes especially in the case of objects. E.g.
sampling on the basis of age groups, sex, or size of the farms.

5. Experimentation.
It is an act of conducting a test or investigation to provide evidence for or
against a theory. For example determine chemical composition of rocks.
Advantages.
a) First hand information is obtained.

b) It gives accurate results when done properly.

c) It may lead to further discoveries.

Disadvantages.
a) It is expensive when it involves use of expensive equipment.

b) It is time consuming.

c) Use of defective instruments may lead to inaccurate answers.

d) Improper handling of equipment may lead to accidents.


Page | 75
Prefer Calling Sir Obiero Amos @ 0706 851 439 for F1-F4 All Subjects Notes
METHODS OF STATISTICAL DATA RECORDING
The information collected should be written down in a systematic and
understandable manner. It involves note taking, drawing field sketches such as
maps, panoramas, transect (cross section), filling in base maps and tables
(tabulating), etc.
a) Note taking. This involves writing notes about what you see in the field
(observation). It is also used during interviews or lectures in the field.
b) Filling in questionnaires. The interviewer may opt to fill answers in the
open-ended questionnaire.
c) Tallying. This is used where measuring is employed as a way of collecting
information. There is the use of strokes which may be vertical or inclined.
A group of five strokes represents an occurrence of five similar items.
d) Drawing Cross sections and maps. This is a way of recording information,
it helps students to recognize and describe relief and land use on the
landscape. Cross sections help students to illustrate relationships
between, slopes, soils, vegetation (physical landscape) and land use
(human activities).
e) Sketching. Information observed in the field about landforms, land use or
a farm may be summarized by drawing a sketch. Sketches should have
the following: Title/ Heading, key (man made feature and physical feature),
and compass direction.
f) Tabulation. Tables can be drawn and the data filled in systematically.

g) Photographing. When photographs taken in the field are processed, the


should be clearly labeled so that they are not mixed up during storage.
Photographing is a technique of data collecting as well as data recording.
h) Tape recording. Conversations and interviews can b recoded on audio
tapes using a tape recorder. This is useful in a case where the interviewer
does not need to keep on interrupting the interview or discussion.
i) Labeling samples. Collected samples are put in polythene bags for safe
keeping and storage. Each of the bags should be labeled clearly.

Page | 76
Prefer Calling Sir Obiero Amos @ 0706 851 439 for F1-F4 All Subjects Notes
STATISTICAL DATA ANALYSIS.
This involves analyzing the data collected through;
a) Calculating percentages. If data has to be compared, proportions are bets
expressed in percentage.
b) Calculating ranges. Raw data can be organized systematically in an array
(ascending order) e.g of age. Range is the difference between the largest
and the smallest number in a set. It is significant when dealing with
temperature and population. For example, Diurnal Range, Mean monthly
range and mean annual range.
c) Measures of central tendency. This involves determining the average values
of set which may reflect the general characteristics. For example;
a. Mean. This refers to average or arithmetical. For example

Mean temp =Daily max temp. + Daily min temp.


2
b. Median. The middle value in a set of statistics.

c. Mode. It is the value which occurs most frequently in a set of data.

STATISTICAL DATA PRESENTATION.


After data analysis, the data is processed by converting the numerical data
into other forms which are not statistical. E.g use of statistical tables, line
graphs, simple bar graphs, combined lie and bar graphs and wind rose.

1) Statistical tables.
Concrete numerical data is arranged in rows and columns according to
categories to which it belongs. E.g numerical values of R/F and temperature
can be shown below.
Temperature and Rainfall for Thika (altitude 1530m)
J F M A M J J A S O N D
TEMP O C 19 21 21 20 18 18 17 17 19 20 20 20
RAINFALL(mm) 55 47 119 207 168 35 27 27 14 75 164 66

Page | 77
Prefer Calling Sir Obiero Amos @ 0706 851 439 for F1-F4 All Subjects Notes
Advantages of statistical tables.
1. Tables showing the actual numerical values. Therefore it is easy to obtain
accurate data directly from them.
2. Geographical statistics can be located more easily facilitating rapid and
accurate reading of data.
3. The data is summarized in a relatively small space.

4. Comparison is easy between various groups.

5. Distinct patterns are revealed within the data.

6. The data is easily understood. The table shows it in a clear and orderly
manner.
7. It is a straight forward technique of publishing statistical data and making
it available for future reference.

Disadvantages.
1. Reading and interpretation of facts may be difficult if there is a large
range of values in the table.
2. Comparing data between rows and columns may be difficult if there are
numerous figure.
3. It takes a longer time to interpret data from a table.

4. Tables are not eye catching.

2) Graphs.
This is a planned drawing consisting of a line, lines or bars showing the
relationship between two or more sets of values.
Statistical graphs are groups of diagrams which are basically concerned with
relationship between varying quantities.
Two kinds of variables exist; independent variables and dependent variables.
a) Independent variables. These are variables which change steadily and
regularly. E.g time and distance, but whose change is not dependent on
the other value. They are plotted on the X-axis.
b) Dependent variables. These are variables that change irregularly and may
be controlled by the independent variables. E.g Temperature, Rainfall and
pressure. They are plotted on the Y-axis.

Page | 78
Prefer Calling Sir Obiero Amos @ 0706 851 439 for F1-F4 All Subjects Notes
Simple Line Graphs.
Line graphs are a drawing on which the dependent variables are represented
by a continuous straight line or smooth curved line.
Curved line graphs are used for continuous data like mean monthly
temperature, population growth rate and changes in air pressure.
Straight simple line graphs with sharp angular turns are used for data which
may have some discontinuity E.G tea and coffee production over a period of
ten years.
Construction of a simple line graph.
a) Draw the X and Y axis on a graph paper.

b) Decide on a suitable scale for both axes. E.g X axis = 1cm represents 1
month and Y axis= I cm represents 5 C
Choose the scale carefully so as to avoid giving exaggerated impression.
c) Plot the values accurately using small faint but visible dots.

d) Join the plotted points using a ruler and a sharp pencil for straight line
graph and a smooth curve for the curved graph.
e) Draw another vertical axis on the right hand to mark the end of
December. The line graph should be touching on both axes.
f) Label both axes and write an informative title for the graph in a clear
space above.
Illustration.
Draw a line graph to illustrate the data below;
Temperature for Thika (altitude 1530m)
J F M A M J J A S O N D
TEMP O C 19 21 21 20 18 18 17 17 19 20 20 20
TEMPERATURE FOR THIKA AT ALTITUDE 1530M

25
TEMP . C
Temp.
in C
ͦ
20

15

10

0
0 J F 2 M A 4M J 6J A 8S O 10N D 12
MONTHS HS-1cm rep 1 month VS 1cm rep 2.5 C

Page | 79
Prefer Calling Sir Obiero Amos @ 0706 851 439 for F1-F4 All Subjects Notes
Advantages.
a. Easy to construct.

b. Easy to read since it shows the trends.

c. They enable easy comparison of variations in the association between


two variables.
d. It does not involve lengthy and tedious calculations.

e. Simple curved line graphs allow for determination of intermediate


values and exact values.
f. The trend of the interrelationships can easily be seen.

Disadvantages.
a. It is difficult to obtain the exact values from the graph especially by
interpolation.
b. Fluctuation of values may be exaggerated out of proportion due to poor
choice of scale.
c. False impression on continuity of date may be given, i.e. on production;
periods when there was no production are not indicated.
d. It does not give precise visual impression on actual quantities.

e. It shows no comparison between two or more sets and data.

Simple Bar Graph.


➢ Here, quantities are represented by a series of columns and is suited for
representing discrete data (where quantities are tangible).
➢ For example, Rainfall data, crop production by weight or area under
crop.
➢ Length of the bar shows the quantity being represented.

Construction steps.
a) Draw the X and Y axis on a graph paper.

b) Decide on a suitable scale for both axes. E.g X axis = convenient width
of bar and separating space and Y axis= bars should not be too long nor
too short.

Page | 80
Prefer Calling Sir Obiero Amos @ 0706 851 439 for F1-F4 All Subjects Notes
Choose the scale carefully so as to avoid giving exaggerated impression.
c) Plot the values accurately using small faint but visible dots placed in
the middle of the top line that will form the end of each bar.
d) Neatly draw the outlines of all bars and label them if necessary.

e) The bars can be shaded uniformly in pencil or may be given a uniform


colour tint.
f) Label both axes and write an informative title for the graph in a clear
space above.

Illustration
Draw a Bar Graph to illustrate the data below; Rainfall for Thika (altitude
1530m)
J F M A M J J A S O N D
RAINFALL(mm) 55 47 119 207 168 35 27 27 14 75 164 66

THIKA: Mean monthly rainfall


250 250
Rainfa ll
in mm
200 200

150 150

100 100

50 50

0 0

J F M A M J 55 J A S O N D

Months

NB; all bars start at zero line (x-axis). All values start at zero (bottom left
hand corner). When the time element is of significance, bars are drawn
horizontally with the longest bar on top. E.g production of a commodity by
various countries. E.g a mineral Advantages.

Page | 81
Prefer Calling Sir Obiero Amos @ 0706 851 439 for F1-F4 All Subjects Notes
a. Easy to draw.
b. They enable easy comparison of variations in the Quantities.
c. It does not involve lengthy and tedious calculations.
d. The trend of the interrelationships can easily be seen/gives clear visual
impression.

Disadvantages.
a. It does not show production of various commodities at the same time
hence comparison is not easy.
b. It does not show variation of production over time.

c. One cannot easily establish the cause of variation of the phenomena


portrayed by the group.
d. Inadequate deduction can be made as a result of scaling constraints.

Combined Line and Bar Graph,


It is simply a combination of the line and bar graphs. It is used to show the
relationship between two types of data especially temperature and rainfall.
Procedure for drawing is similar to that of a simple line and bar graph (both).
Temperature figures are plotted on the right hand side of the Y-axis while
rainfall on the left.

Illustration.
Advantages.
a. Easy to draw and read.

b. It shows relationship between two different types of data.

Disadvantages.
a. It does not show relationship between the same type of data especially
temperature or rainfall amount of more than one place.
b. It is difficult to choose a suitable scale when the value for each variable
differs by a great magnitude.
c. The cause of variation in the variables cannot be established from the
graph.

Page | 82
Prefer Calling Sir Obiero Amos @ 0706 851 439 for F1-F4 All Subjects Notes
This a chart WINDROSE.
in which data on wind direction for a period of time , say a month, is
recorded. The central part is either a circle or an octagon with each side
representing one of the eight cardinal points of the compass.

Steps in construction.
a. Draw an octagon or a circle at the centre of a piece of paper (small in size
about 1cm to 1.5cm in diameter.
b. Choose a suitable scale to represent the data in the frequency table
(length of the bars). It must accommodate the largest and the smallest
frequency.
c. From each side of the octagon, draw a bar at right angles in proportion
to the number of times the wind blew from that side.
d. Calm days number is written at the centre of the octagpn/circle,

e. Draw a sign indicating the four cardinal points of the compass besides
the windrose to help determine direction.
f. All bars must be of the same thickness (sometimes straight lines can be
drawn instead of bars.) g. Draw a linear scale at the bottom .
Illustration.

Wind rose
N

N
S

No. of calm
W days E

Scale 1 day

Advantages.
a. It gives a good visual impression of wind flow.

b. It is relatively easy to construct as it saves time.

c. It easy to understand the information represented.


Page | 83
Prefer Calling Sir Obiero Amos @ 0706 851 439 for F1-F4 All Subjects Notes
Disadvantages.
a. Numerical values are difficult to extract since it involves measuring and
calculating using the scale given.
b. It is difficult to determine the exact day or time when the wind blew from
a particular direction.
c. The pattern of wind flow over a given period cannot easily be seen from
the diagram.

Page | 84
Prefer Calling Sir Obiero Amos @ 0706 851 439 for F1-F4 All Subjects Notes
MAPS AND MAP WORK.
Map work involves drawing pictures, naps and plans.
1. Pictures. These are images of an actual object represented either as a
drawing , painting or a photograph. It can be bigger than smaller or equal
to the object depending on the size of the object and desired size.
2. Maps. These are representations of a part of the earth or whole earth on a
flat surface like a sheet of paper or chalkboard usually drawn to scale. It
is drawn as if the area being represented was viewed from above.
Emphasis is on specific desired items. E.g a physical map emphasizes
mountains, rivers valleys and plains etc.
3. Plan. This is a map of a place or picture drawn to scale for specific use and
giving specific information. It is selective and shows many details. E.g a
plan of a house showing room details, windows and doors.

Distinction between pictures plans and maps.


A picture gives details in their visible shapes and sizes.
A map gives details which a map marker would like to depict and drawn as
if the drawer was above the ground (objects and features are indicated by
symbols).
A plan is a chart drawn and meant to give very minute details about very
small areas like towns or villages. Also, like a map, a plan is drawn as if the
drawer was directly above the ground.

Plan of a classroom

Page | 85
Prefer Calling Sir Obiero Amos @ 0706 851 439 for F1-F4 All Subjects Notes
Main types of maps and their uses.
a) Topographical maps.
Topography is the description of the surface features either natural or
human made. Natural features include rivers, lakes, mountains etc.
manmade features include Houses, towns etc.
Such maps are used in the following ways.
~ They are used by travelers to find way /direction.
~ They can be used to calculate distance to destination.
~ They are useful in locating physical features like landforms.
~ They give us information about distribution of geographical
phenomena like vegetation and land uses.

b) Atlas maps.
An atlas is a collection of maps, usually drawn to scale, in one volume. The
maps cover larger ground on a small sheet and contain a wide range of
information.
Such maps are used in the following ways.
~ Relief maps concentrate on aspects of relief and gives information on
distribution of physical features like mountains, valleys and plains.
~ Climatic maps show information on temperature, rainfall, winds and
atmospheric pressure. They provide information about different
climates of different parts of the world.
~ Vegetation maps show distribution of natural vegetation and provide
this information to geographers.
~ Political maps show political divisions usually countries, regions and
districts. They are used to locate other countries and compare their
shapes and sizes.
~ Population maps show distribution and settlement including towns.
~ Economic maps show distribution of various human activities of
economic significance. E.g farming, transport, mining, livestock
rearing etc.

Page | 86
Prefer Calling Sir Obiero Amos @ 0706 851 439 for F1-F4 All Subjects Notes
c) Sketch maps.
These are maps drawn to give specific information and not usually drawn to
scale. They are not usually proportional to the land area being represented.
A good sketch map should have the following qualities;
~ It must be neat and clear.
~ It must be accompanied by a title.
~ It must be enclosed in a frame.
~ It should have a key to explain meaning
of symbols used.
~ It should have a compass point showing
north.
Uses of a sketch map.
Since they are easily drawn, they are used to store a lot valuable information
which can be summarized.

MARGINAL INFORMATION.
This is the information given in the margin of a map. It gives additional
information besides what is already shown on the map. It tells many things
about the map and includes the following;
a) Map series. This is the number identifying the map sheet with other map
sheets in the same group. It is enclosed in a box. E.g (Y731(D.O.S 423)).
b) Edition. This gives the year the map was published or reprinted E.g
Edition 1971.

Map identification
Series
Sheet Y731 (D.O.S. 423
137/2 and Uganda
83/2 (part of)
Edition 1971
Sheet 137/2
And Uganda Sheet 83/2 (part of)

Page | 87
Prefer Calling Sir Obiero Amos @ 0706 851 439 for F1-F4 All Subjects Notes
c) The sheet index to adjoining sheet. This a box containing nine squares where
the centre square represents the map itself. Index number is shown
identifying the map in relation to adjoining sheets. Index to adjoining sheets
135/1 135/2 136/1
MURANG’A EMBU SIAKAGO
135/3 135/4 135/1
MAKUYU ITHANGA MASINGA
149/2
OL
149/1 DONYO 150/1
THIKA SABUK KANGONDI

d) Copyright. It is symbolized by ©- a law that protects the owners of the


published work.
e) Scale. It is shown as RF-Representative Fraction. E.g 1:50,000 or as Linear
scale with units given in KM or Miles.

LINEAR SCALES
METRES 1 000 0 1 2 3 4 5 6 7 KILOMETRES

MILE 1 0 1 2 3 4 5 6 7 MILES

FEET 2 500 5000 10 0000 15000 20000 25 000 30 000 FEET

f) Compass Direction. It is given in form of a diagram E.g


Compass Direction and Magnetic Variation
Grid North

True North

Magnetic
At Sheet Centre
North 0 32 ’ Magnetic Declination as at January 1991
0 2’
Annual Change:- 5’ West
0 30 ’

This which shows relationship between magnetic North, Grid North and
True North. It states the magnetic variation.
Page | 88
Prefer Calling Sir Obiero Amos @ 0706 851 439 for F1-F4 All Subjects Notes
g) Key. It contains conventional signs and symbols used on the map to
represent both natural and human made features in a given area.
NB; the key is a general one for all maps drawn on that scale. It might
have symbols not appearing on the actual map being read.

MAP SCALES.
A scale is a ratio of the distance on a map to a corresponding distance on the
actual ground. The scale indicates the relationship between the two
distances measured. i.e. one on the map and one on the actual ground
surface. E.G 1:50,000 or 1cm on a map represents 50,000cms on the actual
ground.
The scale of a map is constant for all parts of a map.

Types of scale.
a) Statement scale. This a scale expressed in words. For example, 1cm
represents 1km. in the statement, it means that a distance of 1cm on the
map represents a distance of 1km on the actual ground.
b) Representative Fraction RF. This a scale where distances on a map are
expressed as a fraction of the actual ground distances. E.g 1cm represents
1km can be given as 1/100,000 or in ration form as 1:100,000. The units of
numerator and denominator must be the same. Numerator represents
map distance and denominator represents ground distance.
c) Linear scale. This a line sub-divided into small equal units, each unit
marked on the line representing a distance on the actual ground. It has
the advantage of having ground distances obtained directly without
having to go through tedious calculations. The scale has two sides.

LINEAR SCALES
METRES 1 000 0 1 2 3 4 5 6 7 KILOMETRES

The right hand sight of Zero is the primary scale and the left is the
secondary scale.

Page | 89
Prefer Calling Sir Obiero Amos @ 0706 851 439 for F1-F4 All Subjects Notes
Conversion of scale
1. Statement scale to Representative Fraction. For example 1cm represents 2 km.
this means that 1cm represents 2x 100,000cm. units on both sides in RF
must be the same. 1cm rep 200,000cm = RF-1/200,000 0r 1:200,000.
2. RF to statement Scale. E.g 1/200,000 0r 1:200,000. This means 1cm
represents 200,000cm on the actual ground. 1cm represents
(200,000/100,000) km 1cm represents 2km.
NB; standard unit on map is cm while standard unit on the ground is km.
3. Linear scale to statement scale. If the markings on the linear scale are at
intervals of 2cm but representing 4 km on the ground.
2CM REPRESENTS 4KM WHICH is similar to 1cm rep 2km
4. Linear scale to RF. Use step 3 above then step 1.
5. RF into linear scale. Convert RF into statement. Draw a line and divide it
into 2cm intervals. Mark the 0 point at the 2 nd marking from the left. At
every 2cm interval, write the value of distance which will be as follows;
0, 4, 8, 12, 16,20km. Sub-divide the portion on the left into ten equal parts
and at the end, write the value in metres. I.e. 4000m.

Sizes of scales.
Maps may be the same size on paper but will differ in the area of land each
represents. In order to fit the maps representing different sizes of land on the
same size of paper, it is important to vary the scale sizes.
There are three categories of scale;
a) Small scale maps. They are used on a map when showing a large area of
land on a small size of paper. Such maps show very few details because
of limited scale. For example 1:250,000, 1:500,000 and 1:1000,000.
b) Medium scale maps. They represent a relatively smaller area on a given
size of paper. They fall between small scale and large scale maps. They
range from 1:125,000 to 1:500,000. They show more details when
compared to small scale maps.
c) Large scale maps. They represent a small area of land on a paper. They
show a lot of details of the area represented. For example 1:25,000,
1:50,000, 1:10,000 and 1:2500.
NB; the larger the denominator of the RF, the smaller the scale.
Page | 90
Prefer Calling Sir Obiero Amos @ 0706 851 439 for F1-F4 All Subjects Notes
Uses of scale.
1) Measurement of distances on maps.
a) Measuring distance along a straight line.
(i) Use of a sheet of paper. Mark the two points X, Y on a map and draw a
straight pencil line joining them. Place the edge of paper along the line
and mark the distance xy. Read the distance by placing the edge of the
sheet of paper against the linear scale.
(ii) Using a pair of dividers. Open the pair of dividers so that its two points
are over the two points between which you want to determine the
distance. Carefully transfer the divider onto the linear scale or ruler.
Read off the distance on the ruler or linear scale. If the distance is
longer than the span of the divider, stretch the divider to convenient
distance e.g 5km. measure the distance in portions of 5km and record
the number of portions. Add up and calculate the distance.

b) Distance along a curved line. Distance on a winding route for example a


road or river.
(i) Use of a piece of string. Place one end of the string at A and mark this
point on a string using ink. Lay the string carefully along the curved
line until you reach B. Mark B on the string and place the string against
the linear scale to obtain the distance.
(ii) Using a piece of paper. Fold a paper to obtain a straight edge. Mark the
starting point on the paper. Place it against the starting point A of the
distance on the map. Hold the paper in place with the needle point of
the dividers at the starting point. Rotate it until it covers a strait portion
and
mark. Hold the paper firmly at the new point and rotate to the next
straight portion until all straight portions are transferred onto the paper.
Transfer the edge of paper onto the linear scale to read the distance.

Page | 91
Prefer Calling Sir Obiero Amos @ 0706 851 439 for F1-F4 All Subjects Notes
2) Calculation of areas on maps.
a) Regular shapes. Use standardized mathematical formulae. E.g L x W for
squares and rectangles.
b) Irregular shapes. For example, areas of forests, lakes, plantations, national
parks etc.
(i) Use of rectangles and triangles. Divide the area into a rectangle and
triangle and measure their dimensions. Convert into actual ground
distance using linear scale. Find the areas of rectangle and triangle.
Add up the areas. The total gives the area of the irregular shape.
(ii) Use of strips. Cover the entire area with strips of convenient width (the
area left out should be in balance with the area included). Measure the
dimensions of the strips. Convert into actual ground distance. Find the
area of each strip. Add the area of all the strips to obtain the area of the
required irregular objects or shape.
(iii) Use of Grid square method. The entire map on a 1:50,000 map is covered
with a network of grid squares with each square being 1km² on the
actual ground. Count the complete squares and number them. Count
the incomplete squares as half squares and divide by two to obtain full
squares. Add all the complete squares. The total gives you the number
of squares covering the shape.

Page | 92
Prefer Calling Sir Obiero Amos @ 0706 851 439 for F1-F4 All Subjects Notes
FIELDWORK
INTRODUCTION
Fieldwork is a scientific approach through which geographical knowledge
and skills can be acquired practically in the field. The field is the major
source of primary geographical information (data).

Types of field work.


1. Field excursion.
This is a field trip, study trip or a study tour where students are taken out of
school to visit areas in the vicinity or places far away.
It serves the function of;
a) Reinforcing what is learnt in the classroom with practical experiences
by seeing and observing.
b) Helping to gain more geographical knowledge of the area visited.

c) It helps in identifying and appreciating the existence of geographical


features, human activities and the scenery.
d) It helps in Identifying problems of geographical interest which may
form a subject for discussion at a later stage.

2. Field research.
This is A more advanced fieldwork involving problem solving approach. For
example, by a medical team concerned with a disease outbreak.

3. Field study.
It is a study conducted within the neighbourhood and involving group or
individual collecting information to achieve pre-set objectives. Data
collected is analyzed and conclusions drawn.

Geographical significance of fieldwork.


The findings or results may help us to understand the geography of the area
in the following ways:
• Update the information about the area of study (current situation).
Highlight the new problems facing the people in the area (threats).
Page | 93
Prefer Calling Sir Obiero Amos @ 0706 851 439 for F1-F4 All Subjects Notes
• Shows new solutions and prospects of developing the area
(opportunities).
• ·Help in the understanding of geographical relationship existing in the
area.

Importance of fieldwork.
a) Field work helps students to be observant, to select relevant information,
to record accurately what has been observed and to draw meaningful
conclusions from the observations..
b) It Breaks classrooms monotony of memorizing and listening to the
teacher by providing practical situations.
c) Fieldwork Makes study of geography real by enhancing what has been
learnt in the classroom. This is achieved through seeing real objects in
their natural settings and studying them.
d) Fieldwork within the school environment Encourages students to
appreciate their environment.
e) It Enables learners to get first hand information from the field.
f) It enables one to apply the knowledge leant in the classroom to real life
situations.
g) Fieldwork encourages critical thinking or enquiry and gives students a
sense of personal achievement.
h) Due to exposure in the field, the learners are assisted in understanding
problems on a larger scale, e.g those affecting a locality, a country or a
region.
i) Fieldwork enables us to understand the relationship between physical
and human geography. In reality.

FIELDWORK PROCEDURE
Preparation of Fieldwork
For any field work study to be undertaken, both technical and organizational
decisions should be considered. These include the following:
a) Identifying the topic to work on.

b) Identifying the area where to carry out fieldwork.

Page | 94
Prefer Calling Sir Obiero Amos @ 0706 851 439 for F1-F4 All Subjects Notes
c) Setting up the objectives of the study
d) Formulation of hypothesis.
e) Preparation for the field study.
f) Actual field work.

Step 1: Identify the topic


The topic of study is an important guide, gives direction in carrying out
field work. It is sometimes referred to as the problem. First identify the
subject matter. Design the topic to establish the scope (what to study).
Identify the area of study (where to look for data from)
Think of the most informative and appropriate area for the topic. It can be
even the current topic being covered in class. The teacher finds suitable
study area. E.g. A study of rocks around the school.

Step 2: Identifying the area where to carry out fieldwork.


The area where carry out the study has to be chosen carefully to ensure
success of the study. The area must contain sufficient information and
within a convenient distance.

Step 3: Set specific objectives of the study


Objectives summarize what is to be achieved by the study and should
address key factors of the topic. Students are led to suggest short and precise
instructions to facilitate the collection of data in the field. They should be in
a logical sequence. The objectives are intended to direct the student’s
activity towards acquiring specific skills. Objectives should be stated in
action verbs that are specific enough to be achieved. A standard objective
should start with the words “to find out” “to investigate”, “to identify”
Examples:

Page | 95
Prefer Calling Sir Obiero Amos @ 0706 851 439 for F1-F4 All Subjects Notes
ACTION AVOID VAGUE VERBS (ABSTRACT)
VERBS
To: - To: -
• Identify • Know

• Find • Appreciate

• Determine • Understand

• Investigate • Study

• Calculate These vague verbs do not direct student’s activity


• Analyze, etc. towards acquiring specific data.

Objectives are exact statements of the activity and what is supposed to be


observed.

The objectives of the study of Kimilili market are as follows:


a) To locate Kimilili market.

b) To draw a sketch map showing site of the market.

c) To explain factors for establishment of the market.

d) To identify problems facing the market in the contemporary period.

e) To find out how the market is dealing with these problems, etc.

Step 4: formulation of hypothesis.


A hypothesis is an idea, guess or suggestion from which reasoning or
explanation is sought. Or it is a statement of an outcome which is
anticipated. It can be a tentative answer or the proposed solution to the
problem which the study is designed to test or prove. A hypothesis can be
stated in a declarative or substantive form. E.g. the majority of the goods sold
in the market are vegetables.
A hypothesis can be stated in positive form or in a negative form (null
hypothesis). It can also be stated in a question form. A good hypothesis
should be simple but not obvious. It must leave room for acceptance or
rejection of the suggestion.

Page | 96
Prefer Calling Sir Obiero Amos @ 0706 851 439 for F1-F4 All Subjects Notes
Step 5: preparation for the field study
These are the preparations you will make once you decided to carry out the
field study. The kind of preparation you make depends on the type of
fieldwork you intend to make. The main purpose of planning is to take
note of points and activities that would be relevant for the fieldwork
exercise such as;
a) Seek for official permission to carry out fieldwork from relevant
authorities, i.e. school administration and administration officials of the
area of study.
b) Conduct a reconnaissance.

c) Hold discussions in class.

d) Decide on the methods of data collection.

e) How much time to spend on each activity/prepare a work schedule.

f) Prepare questionnaires

g) Make budget for fieldwork exercise if there is need and pass it on to


relevant authorities.
h) Fix the date for fieldwork and a program for fieldwork to be put in
place.
a) re-connaissance/Pilot study
Pilot study is a pre-survey, or collecting of preliminary information. I.e.
students and teachers visit the area of study well in advance;
a) To familiarize with the area and save time during the actual study.

b) To judge and gauge how the actual fieldwork could be conducted.

c) To gather general information and relevant documents from officials.

d) To help one in deciding on the appropriate methods of data collection.

e) To determine the appropriate routes to be taken.

f) Helps in assessing the suitability of the area as a source of the


information you require.
g) Helps in identifying the problems that are likely to be experienced.

h) It is possible to assess the cost of the study and plan for it appropriately.

i) It helps in formulating relevant objectives and hypothesis. They can be


revised after the pre-survey.
j) It helps in general planning and preparation of a work schedule.

Page | 97
Prefer Calling Sir Obiero Amos @ 0706 851 439 for F1-F4 All Subjects Notes
b) Class discussions.
This are held after the reconnaissance. The type of data to be collected is
identified and a decision on the suitability of the formulated objectives and
hypothesis is made. Adjustments, if necessary are made.
Methods of data collection are decided at this stage.

c) Determine the different methods (skills) to be used in fieldwork to collect data


Methods should be relevant to the topic of study and objectives. Explain the
procedure taken to use the method. Information (specific data) obtained by
the using a given method should be brought out.
Some of the basic commonly used methods in geographical
fieldwork studies include: a) Direct observation.
b) Recording.

c) Map orientation and reading.

d) Interviewing.

e) Sampling.

f) Pacing.

d) Identify the tools and equipment to be used in the collection of data.


Examples: Stationary – pens, pencils, and clip boards, note book, base
map. Measuring equipment- tape measure, foot-ruler, and magnetic
compass. e) Preparation of a work schedule.
A work schedule is a timetable to be followed on the day of the field study.
It gives a step by step plan of activities for the day indicating the specific
times when each activity should take place.
Such a schedule is important in the following ways;
a) It ensures proper time management and reduces the tendency of wasting
time.
b) It ensures that no important area will be inadequately covered or
forgotten.
c) It ensures that the one carrying out the study remains on course. i.e does
not deviate into irrelevant areas.
d) It is a pointer as to how much time will be required for the study.
Page | 98
Prefer Calling Sir Obiero Amos @ 0706 851 439 for F1-F4 All Subjects Notes
e) Briefing (coordination) and dividing in groups.
This specifies the details on how you are going to operate (operational plan).
This involves telling students what is expected. It involves the following: -
a) Reviewing the topic to work on.

b) Revising the objectives of the study, specifying how individual


objectives are to be achieved.
c) Describing the methods of recording data.

d) Emphasizing how to approach people in a polite way to maintain good


relationship.
e) Grouping the students into working teams (team work) and Assigning
students different responsibilities to do.

Step 6: Actual fieldwork (collecting relevant data)


This is the stage of collecting data practically, the students go in the field
and apply the knowledge and methods, identified and discussed in the
pre- fieldwork preparations to collect the geographical data about the area.
Note that:
a) .In the field, the first thing to do is to introduce /notify your presence to
relevant authorities or management by producing the letter that you
used to seek permission during your pilot study.
b) The best way to start fieldwork is identifying a strategic position where
you can see all you want to study.
c) Periodically, review the topic and objectives to ensure that data being
collected is relevant.

Step7: Follow up activities (analyzing data and presenting results of fieldwork)


The main purpose of follow up exercise is to re-organize and discuss results,
concerning the topic and the objectives of fieldwork as spelt out from the
beginning.
The following are done as follow-up activities;
a) Students discuss and compare data (is sharing information through
discussion).
b) Polish up diagrams and sketches drawn during the study.

Page | 99
Prefer Calling Sir Obiero Amos @ 0706 851 439 for F1-F4 All Subjects Notes
c) Organize the data, analyze and interpret the data and show the different
relationships.
d) After compiling each group reports their findings (data presentation).
e) Students write a report on the entire fieldwork exercise.
f) Report writing (this is data analysis according to objectives)

Methods of data collection.


1) Observation.
Direct observation is to see and identify patterns and interrelationships.
This method gives opportunity to record what is seen directly. The method
serves well in a situation where the phenomena being understudied cannot
communicate back. E.g. rocks, soil, rivers and vegetation.
Advantages of the observation technique.
i. The data obtained is reliable since it is first-hand information.

ii. One collects only what is relevant to the study. iii. It is time
saving because you do not have to look for data in many places.
Disadvantages.
i. It has an element of subjectivity because the data may be the findings of
only one person’s observations. ii. The choice of spots to gather data from
may be an individual’s choice and preference, which may be biased.
iii. Data on past activities may not be available during the study.

iv. Where the required information may have changed with time, one can
make wrong conclusions.
v. Visual disabilities may reduce the effectiveness of the technique. vi. It
involves a lot of traveling which makes it tiresome or expensive.

2) Interviewing.
Interviewing is when questions are asked to respondents and answers
received from them to obtain information about desired geographical
aspect. An interview may be conducted face to face or on telephone.
Below are some of the guidelines on how to achieve a conducive
atmosphere for an interview;

Page | 100
Prefer Calling Sir Obiero Amos @ 0706 851 439 for F1-F4 All Subjects Notes
a) Approach people politely.
b) Create a warm and friendly atmosphere for respondents.

c)Assure the respondents that the information they give remains


confidential.
d) Avoid unnecessary interruptions during the interview.

e) Do not suggest answers. Let all the answers come independently


from the respondents. Advantages of the interview technique.
a) It gives first-hand information through direct responses from the
resource persons.
b) The interviewer can seek clarification from the respondent in case of
ambiguity in answers.
c) The interviewer is free to seek for more information by initiating further
discussions or by asking other questions.
d) Information can also be obtained from people who cannot read or write.

e) The interviewer has the opportunity to create a good rapport with the
respondent and this would ensure that reliable answers are given.
f) The interviewer can also gauge the accuracy of the responses.

Disadvantages.
a) It is time consuming since the interviewer has to handle one person at
a time.
b) It is expensive and tiresome since the interviewer has to move to
different places to meet the respondents.
c) It can easily be prone to language barrier where the interviewer and the
respondent do not speak the same language. Use of interpreters might
distort some information.
d) Where resource persons may have forgotten, information on events in
the past may be unreliable.
e) The respondent may sometimes through exaggeration give wrong
information or even deliberately mislead.

Page | 101
Prefer Calling Sir Obiero Amos @ 0706 851 439 for F1-F4 All Subjects Notes
3) Administering questionnaires.
This is a system of data collection where the researcher gives out a rigid
questionnaire to the people in the field. The questions are prepared in
relation to the objectives and hypotheses of the study.
There are two types of questionnaires;
a) The open-ended questionnaire.

This questionnaire has questions which are not limiting. The questions
act as a guide to a person carrying out the field work. He/ she can go
even beyond the question requirement when answering the questions.
Such a questionnaire is also called a personal interview questionnaire.
b) The rigid questionnaire.

This has a series of questions which are administered to a respondent in


the field. In this questionnaire, possible answers may also be given
within the questions. The responds only ticks the correct answer.

Characteristics of a good questionnaire.


a) It should have questions in a simple language.

b) The questions should be short and clear.

c) They should be arranged in a logical sequence.

d) The questions should be relevant to the field study.

e) Questions that touch on a respondent’s privacy should be avoided.

f) Questions should be objective and devoid of bias.

Advantages of using a questionnaire.


a) Comparison can be made easily since similar questions are used for all
respondents.
b) The data obtained is first hand.

c) The physical appearance of the conductor of the field study is not


necessary since questions are straight forward.
d) A lot of information can be collected within a questionnaire if the
questions are well prepared.
e) It saves time since all respondents are handled at the same time.

Page | 102
Prefer Calling Sir Obiero Amos @ 0706 851 439 for F1-F4 All Subjects Notes
Disadvantages.
a) Some respondents may write down inaccurate answers thus rendering
the data inaccurate.
b) Data analysis may become difficult in a case where different answers
are obtained for the same question.
c) Some questionnaires may be returned unfilled or not returned at all in
case of a posted questionnaire. This may be in case of lazy or
uncooperative respondents.
d) Where there is a language barrier or in case of illiteracy, the technique
would not be useful.

4) Content analysis
This is a technique employed to collect data from secondary sources like text
books, magazines, newspapers, census reports, statistical abstracts, atlases
etc.
The techniques employed in content analysis include;
a) Reading and extracting relevant information.

b) Watching films.

c) Viewing photographs.

d) Listening to audio tapes using tape recorders. Advantages of content


analysis.
a) It is easy to get data especially if it is already analyzed.

b) It is cheap because it does not involve extensive traveling.

c) It saves time since all the required information may be obtained in one
place like the library.
d) It is possible to get old data because it is already stored in various
sources.
Disadvantages.
a) It may be difficult to verify the accuracy of available data.

b) The data may be irrelevant to current trends because it is out of date.

c) Up-to-date information may not be readily available.

Page | 103
Prefer Calling Sir Obiero Amos @ 0706 851 439 for F1-F4 All Subjects Notes
5) Collecting samples.
Where there may be necessity to collect more information on items like
rocks, crops, vegetation and soils, than what may be gathered in the field,
their samples may be collected for further testing in laboratories. 6)
Measuring.
Where it might be necessary to gather information on distances, heights,
areas, or depths, measurements may be taken and the results recorded.
Sometimes estimations may be taken where accurate measurements are
impossible.

7) Pacing
Pacing is a method of measuring using one’s stride.

What to note
The strides made should be equivalent to a specific unit for
example metre foot. Strides are therefore used to estimate sizes
and distances.

8) Photographing
Some information in the field may be captured on film or video using
cameras.

9) Sampling
A sample may be regarded as a specimen or a small portion of a whole or a
bigger population, or part of the area under study taken to show what the
rest is like. For example, it could be a percentage of total population to be
interviewed or soil sampled rock type etc.
Sampling method is useful when a questionnaire is used and when it is
impossible to get the views of everybody. Where the area under study is
too large, the selection of a representative sample (10% of the total
population) is necessary.

Page | 104
Prefer Calling Sir Obiero Amos @ 0706 851 439 for F1-F4 All Subjects Notes
Types of sampling techniques.
a) Random sampling. It is the technique that involves choice of places to be
used as bases for study randomly. For example through dividing the
study area into imaginary squares using lines then picking the study
area through some kind of raffle.
b) Systematic sampling. This method is applied where distribution of
phenomenon is even. E.g. a forest of cypress trees.
c) Stratified sampling. In this method, the study samples are selected
according to their groups or classes especially in the case of objects. E.g.
sampling on the basis of age groups, sex, or size of the farms.

Methods of data recording


The information collected should be written down in a systematic and
understandable manner. It involves note taking, drawing field sketches such as
maps, panoramas, transect (cross section), filling in base maps and tables
(tabulating), etc.
a) Note taking. This involves writing notes about what you see in the field
(observation). It is also used during interviews or lectures in the field.
b) Filling in questionnaires. The interviewer may opt to fill answers in the
open-ended questionnaire.
c) Tallying. This is used where measuring is employed as a way of
collecting information. There is the use of strokes which may be vertical
or inclined. A group of five strokes represents an occurrence of five
similar items.
d) Drawing Cross sections and maps. This is a way of recording
information, it helps students to recognize and describe relief and land
use on the landscape. Cross sections help students to illustrate
relationships between, slopes, soils, vegetation (physical landscape) and
land use (human activities).
e) Sketching. Information observed in the field about landforms, land use
or a farm may be summarized by drawing a sketch. Sketches should
have the following: Title/ Heading, key (man made feature and physical
feature), and compass direction.
f) Tabulation. Tables can be drawn and the data filled in systematically.
Page | 105
Prefer Calling Sir Obiero Amos @ 0706 851 439 for F1-F4 All Subjects Notes
g) Photographing. When photographs taken in the field are processed, the
should be clearly labeled so that they are not mixed up during storage.
Photographing is a technique of data collecting as well as data
recording.
h) Tape recording. Conversations and interviews can b recoded on audio
tapes using a tape recorder. This is useful in a case where the
interviewer does not need to keep on interrupting the interview or
discussion.
i) Labeling samples. Collected samples are put in polythene bags for safe
keeping and storage. Each of the bags should be labeled clearly.

Methods of data analysis.


This involves analyzing the data collected through;
a) Discussing the findings from the field.

b) Individuals or groups giving their reports through group leaders.

c) Calculating percentages means medians and modes.

d) Putting data into groups or categories.

e) Data classification.

f) Laboratory testing of samples carried from the field.

Methods of data presentation.


Analyzed data can be presented in the following ways;
a) Writing reports in essay form.

b) Drawing tables, graphs and charts.

c) Drawing maps to show the location or distribution of phenomena.

d) Displaying photographs. Displaying samples such as those of soil


types, rocks and crops Problems encountered in the field.

1) Language barrier. This is a common problem especially when using the


interview and questionnaire techniques. A lot of data may not be
collected if the interviewer and the interviewee do not speak the same
language. Where the respondent deploys an interpreter, a lot of
information may be distorted.
Page | 106
Prefer Calling Sir Obiero Amos @ 0706 851 439 for F1-F4 All Subjects Notes
2) Illiterate respondents. Some respondents may not understand the questions
because they are illiterate.
Where they attempt to answer the questions, they may give wrong
information.
3) Hostile people. Owners of farms may be unhappy with trespassers. Those
to give information may be unwilling thus making then fieldwork
unsuccessful.
4) Lack of data. Sufficient data may be lacking or incomplete in the selected
areas.
5) Bad weather. Weather can change while in the field, to become very hot.
Heavy rains may also interfere with the study.
6) Accidents in the field. This may happen due to poor handling of equipment.
The researcher can also fall thus getting injured.
7) Noise. E.g. in market places
8) Attack by wild animals, especially where the study is carried in bushy areas.
E.g. snakes and insects.
9) Inaccessibility of some areas. Some barriers like swamps, muddy areas,
rivers, steep slopes, deep slopes and deep valleys may hinder accessibility
to vital information.
.

Page | 107
Prefer Calling Sir Obiero Amos @ 0706 851 439 for F1-F4 All Subjects Notes
MINERALS AND ROCKS
Minerals
Minerals are inorganic substances which occur naturally at or beneath the
surface of the earth.
Most Minerals are formed from more than one element and majorly from
oxygen, potassium, magnesium, iron, aluminum, sodium, calcium and
silicon.
Others like diamond, gold and silver are formed from only one element..
Minerals like sulphur have no crystals.
Characteristics of minerals.
1) They have different degrees of hardness. talc is the softest while industrial
diamond is the hardest.
2) Some minerals aggregate into distinct shapes.
3) Some minerals like gold, silver, copper and diamond have only one
element while others like bauxite have more than one.
4) Minerals can be opaque, translucent or transparent.
5) Minerals have different textures (feel).
6) Minerals have specific colours, for example, gold is yellow and copper is
brown.
7) Minerals have luster. This refers to the surface appearance of a mineral as
it reflects light.
8) Minerals have different degrees of tenacity. they can be described as
brittle, elastic, ductile or flexible.
9) Minerals differ in streak. This is the Colour that a mineral leaves when it
is rubbed against a hard surface.

Rocks.
A rock is a substance that is an aggregate of mineral particles. It may be made
of particles of one mineral only or a combination of two or more minerals.

Page | 108
Prefer Calling Sir Obiero Amos @ 0706 851 439 for F1-F4 All Subjects Notes
Classification of rocks
Rocks are classified according to their mode of formation and appearance.
Three rock types exist;
1) Igneous rocks

2) Sedimentary rocks

3) Metamorphic rocks

Igneous rocks are produced when molten magma cools and solidifies.
The magma may solidify within the earth’s crust forming intrusive igneous
rocks or on reaching the earth’s surface forming extrusive igneous rocks.
Some igneous rocks like granite cool slowly forming large crystals. Others
like basalt cool rapidly and thus contain small crystals.

INTRUSIVE ROCK TYPES


Intrusive rocks crystallize from magmas that have been intruded into the
earth's crust at depths far below the surface. Intrusive rocks that were
formed deep in the earth's crust are called plutonic rocks. They are generally
coarse grained (mineral grains greater than 1 millimeter in diameter).
Those formed near the surface are called hypabyssal rocks. They can be
found in features like dykes and sills. Examples are dolerite, Porphyrite,
porphyry, Diabase, lamprophyre and Granophyre.
Intrusive rocks examples;
a. Gabbro. It is a basic rock that contains a high %-age of ferromagnesian
minerals like pyroxene, olivine, plagioclase feldspar and about 25 to
50% silica.
b. Diorite is an intermediate rock containing plagioclase feldspar,
hornblende or pyroxene.
c. Granite is an acidic rock, containing at least 65% silica with dominant
minerals being potash feldspar, quartz, hornblende, biotite (black mica).
d. Peridotite. It is an Ultrabasic rocks almost completely composed of
olivine and pyroxene.

Page | 109
Prefer Calling Sir Obiero Amos @ 0706 851 439 for F1-F4 All Subjects Notes
EXTRUSIVE/VOLCANIC ROCKS
These rocks are formed on the surface of the earth when lava solidifies. The
kind of rock is largely dependent on the type of lava. Since the lava solidifies
quickly, on falling onto the surface, the crystals formed are very small. The
rocks are fine textured.
They are of two types;
Volcanic ejecta
The materials thrown out from the ground during volcanic eruptions
include solid ash and semi-liquid materials. The most common rock formed
from cooling of these materials is pumice which is spongy and can easily
float on water. When volcanic ash and dust settles on the ground, it may be
compressed to form a rock known as tuff, which is a mechanically formed
sedimentary rock but whose forming materials are of volcanic origin.
Lava flows.
These are formed from the magma that has reached the surface and flown
for a considerable distance before solidifying, depending on its degree of
fluidity.
Basalt is the commonest type of rock formed from this lava. It is a dense black
rock that is similar to Gabbro.
Another rock is obsidian which is also black and has a glassy appearance
and is known as volcanic glass. It does not have crystals due to rapid cooling
and solidification.

SEDIMENTARY ROCKS
These are layered or stratified rocks formed from weathering, erosion,
transportation and final deposition of sediments or particles of other rocks
either on land or in water.
The process whereby sediments or rock particles are deposited by water,
wind or moving ice is known as sedimentation.
Sedimentary rocks are non-crystalline and contain fossils. They are also
known as stratified rocks. There are three classes of sedimentary rocks.
1) Mechanically formed.

2) Organically formed.

3) Chemically formed.

Page | 110
Prefer Calling Sir Obiero Amos @ 0706 851 439 for F1-F4 All Subjects Notes
1) Mechanically formed sedimentary rocks.
They are Form from the compaction and cementation of sediments which
have been laid down in water or on land. The sediments are mainly derived
from the weathering and breakdown of rocks. The rock types are sometimes
reffered to as Clastic sedimentary rocks
They are classified according to the grain size and the kinds of rock
fragments that make up the sediment into arenaceous, argillaceous and
rudaceous rocks.
a) Rudaceous /Coarse-grained rocks. Breccia, Conglomerate and boulder clay.

b) Argillaceous/ fine-grained rocks. Shale, Siltstone, Mudstone, loess and


claystone.
c) Arenaceous mainly sand particles. Weathered quartz or sandstone, grit

2) Chemically formed sedimentary rocks.


They result from chemical processes, generally under water. Examples;
a) Carbonates. These are sedimentary rocks made up of carbonate
compounds. They are formed by the precipitation of calcite from
seawater and also in caves as travertine in form of stalactites and
stalagmites and Trona which is hydrated sodium carbonate.
b) Sulphates and chlorides. They are Also termed evaporates. Sulphates are
formed from the evaporation or desiccation of sulphate compounds. The
rocks form mainly in shallow desert lakes. E.g gypsum which is a
hydrated calcium sulphate
Chlorides. They are formed by partial to complete evaporation of
seawater in enclosed basins. For example rock salts-mainly sodium
chloride.
c) Silicates. Formed when silica in sea water is deposited on the sea bed
followed by accumulation and compaction. E.g. Flint. This is a hard,
glassy sedimentary rock composed of silica that precipitated from water.

Page | 111
Prefer Calling Sir Obiero Amos @ 0706 851 439 for F1-F4 All Subjects Notes
d) Ironstones. Deposition of iron oxide in water followed by compression
leads to formation of rocks such as limonite(common iron ore) and
hematite( a red ferric oxide)

3) ORGANIC SEDIMENTARY ROCKS.


Rocks formed by the accumulation of organic material derived from
remains of dead plants and animals such as polyps.
Organic sedimentary rocks are further classified into;
a) Calcareous rocks. Formed from hard parts of shells and skeletons of
marine creatures. For example limestone and chalk.
b) Ferruginous rocks. Formed from precipitation of hydrated iron oxide
from water. E.g. ironstone.
c) Siliceous rocks. They are formed from organisms whose skeletons are
rich in silica. E.g. Diatomite. They are found in places that were once
under water.
d) Carbonaceous rocks. This are Rock composed of lithified plant
material. E.g. Coal has as its major component accumulations of
organic remains from plants.

METAMORPHIC ROCKS
When rocks are subjected to high pressures and temperatures, the textures
and mineral compositions begin to change. Metamorphism is the
transformation of a rock mass into a rock of generally the same chemistry
but with different textures and minerals. This process occurs particularly
during earth movements.
Types of Metamorphism
There are two major kinds of metamorphism: regional and contact.
a) Regional metamorphism/dynamic/kinetic metamorphism. This
metamorphism involves rocks being exposed to high pressures and
temperatures.
b) Contact metamorphism/thermal metamorphism. It is the process by
which the country rock that surrounds a hot magma intrusion is
metamorphosed by the high heat flow coming from the intrusion. The

Page | 112
Prefer Calling Sir Obiero Amos @ 0706 851 439 for F1-F4 All Subjects Notes
zone of metamorphism that surrounds the intrusion is called the
metamorphic aureole (or halo)

Metamorphic Rock Types


a.Gneiss formed from granites.
b. Hornblende formed from augite.

c. Slates. Formed from clay and mudstone

d. Marble formed from limestone

e. Quartzite formed from sandstone.

f. Graphite formed from coal.

Nb; metamorphic rocks tend to be harder than the original rock, hence more
resistant to erosion. Some metamorphic rocks can further be subjected to
heat or pressure to result in other forms of metamorphic rocks. E.g. graphite
changes to crystalline diamond.

Age of rocks.
The following methods may be used to estimate the age of rocks:
1. Relative dating. This is an estimation of the age of rocks in relation to each
other. (Deciding which rock is older or younger than the other.)
Generally, older rocks underlie young ones. However not all underlying
rocks are older than the overlying ones due to influence of earth
movements like the case of recumbent folds.
2. Absolute dating. The method involves the counting of laminae in some
rocks such as shale. Laminae are the thinnest layers of deposits. The
method is similar to that of counting the tree-rings in very old trees.
3. Radio-carbon dating. This involves the application of the lead-ratio
principle. However it only applies to young rocks.

Distribution of rocks in Kenya.


• Igneous and sedimentary rocks are the most dominant types of rocks in
most parts of Kenya.
• Regions that have been widely affected by Vulcanicity have volcanic
rocks.

Page | 113
Prefer Calling Sir Obiero Amos @ 0706 851 439 for F1-F4 All Subjects Notes
• Those areas that have not been affected by Vulcanicity are covered by
sedimentary rocks.
• Below the surface, there exists a complex basement system of rocks
which is of an ancient metamorphic nature.

Regional distribution of rocks in Kenya.


Central, northern and rift valley regions
• These are areas that have been affected by Vulcanicity and therefore the
main rock type is volcanic.
• In southern Turkana, however, metamorphic rocks are common. Basalt
rocks are found in Nyandarua ranges.
• Sedimentary rocks are found in the inland lake basins like Lake
Naivasha, lake Magadi, and Lake Nakuru.

The coast and north-eastern Kenya.


• The coast is characterized by sedimentary rocks particularly from
Shimoni on the southern coast to Kiunga on the northern coast.
• In the north-east of Kenya, a series of sandstones and siltstones are
common.
• Along the coast particularly near the shore, limestone is the major rock.
Gypsum rock is found in a small section of Malindi.
• Fringing coral reef rocks extend along the shore from the south to the
north.
• Quartzite rocks are found in some parts of Tana River, Kilifi and Kwale
Counties.

Western Kenya and nyanza.


• The regions have the oldest rocks in Kenya and mainly granite and
gneiss. Sedimentary rocks are found in the lake region due to river
deposition.
• In central and northern Nyanza, alternating bands of sandstone and
mudstone are found. Around the Mount Elgon region, volcanic rocks are
found.

Page | 114
Prefer Calling Sir Obiero Amos @ 0706 851 439 for F1-F4 All Subjects Notes
Eastern Kenya.
• The region mostly has metamorphosed rocks of varying origin. For
example schist and gneiss in parts of Kitui, quartzites and marble in parts
of Machakos.
• Some slightly young rocks are also found in these areas. The Yatta
plateau is a volcanic outcrop with volcanic rock. Significance of rocks.
a) Rock weathering produces soils that are important in the growth of crops

like coffee, cotton and sugarcane. These crops have great commercial
values.
b) Rocks aid in storage of underground water, important in water circulation

and formation of springs that are important for human water supply.
c) Some rocks can be used for fuel. E.g. coal. Even mineral oil and gas

formation is associated with sedimentary rocks.


d) Various salts are obtained from rocks. Tunisia and morocco for example

have the largest deposits of rocksalt.


e) Granites and limestones are used for building purposes. Limestone is also

used as a raw material in cement manufacturing. Rocks like granite,


dolerite and Gabbro are used in road construction.
f) Some rocks form fascinating features that are tourist attractions. For

example the crying stones in western Kenya at ileho.


g) Minerals occur in different rock types. For example gold, copper, lead, tin,

silver, zinc, diamond, aluminum, manganese etc. which are used in various
ways.

Page | 115
Prefer Calling Sir Obiero Amos @ 0706 851 439 for F1-F4 All Subjects Notes
MINING
Mining is the extraction of valuable minerals or other geological materials
from the earth from an ore body, lode, vein, seam, or reef.
Mining techniques can be divided into two common excavation types:
surface mining and sub-surface (underground) mining.
Occurrence of minerals.
Minerals and mineral ores may occur in four
main formations; 1. Veins and lodes.
A vein is a crack or a crevice in a mass of rock containing minerals deposited
in crystalline form. If the deposit is large, it is known as a lode. If the veins
and lodes are exposed to the surface, they are known as reefs. They are
associated with igneous and metamorphic rocks. Veins and lodes are
common in metamorphic aureoles of igneous rocks. They are formed when
minerals in molten form are intruded into cracks and as they cool, they
solidify.
Metals that occur in veins include copper, tin, lead and zinc. Unwanted
material after extraction of metals is known as gangue.
2. Beds and seams.

Coal and other minerals may occur in beds or layers as a result of deposition,
accumulation and concentration in horizontal strata of the earth’s crust, of
plant remains. Also potash salts, common salt, and gypsum after formation
by evaporation of water from lakes and ponds may be covered by other
materials so that they occur as underground seams.
3. Alluvial deposits.

Minerals resistant to weathering may be eroded in small particles from veins


of rocks. They then are carried by streams and rivers to the valleys or plains
where they are deposited.
The alluvial deposits of minerals occur within the sand, clay and gravels.
Examples are Gold, tin and platinum.
4. Weathering products.

Some minerals may be formed through deep weathering of a variety of rocks


under tropical conditions of alternating wet and dry conditions. For
example, Bauxite is formed by concentration of aluminium minerals as a
result of leaching by ground water.
Page | 116
Prefer Calling Sir Obiero Amos @ 0706 851 439 for F1-F4 All Subjects Notes
FACTORS AFFECTING EXPLOITATION OF MINERALS
The occurrence of minerals does not guarantee their exploitation. Many
factors must be considered before actual mining begins, some of them are
listed below:
1) Value of the mineral: a mineral is not worth mining unless it has a high
commercial market value. Minerals like gold, diamond, copper and
uranium are often mined at very high cost, but their value and
profitability are just as high.
2) Size and grade of the deposits: unless the deposits are of reasonable size and
high grade, it might not be worthwhile investing in expensive equipment
and basic infrastructure to undertake the mining.
3) Mode of occurrence: if the deposits occur too deep in the earth's crust or in
many remote places such as in the ocean, the cost of extraction may be
too high.
4) Transport costs: minerals occurring in remote, sparsely populated
regions, away from the urban and industrial markets, may not be worth
mining as the cost of constructing transport networks might outweigh
the profit derived.
5) Labor supply: workers must be available if any mining is to be undertaken,
if amine is located in a distant place, it will be very difficult to secure a
reasonable supply of workers and site engineers.
6) Level of technology. This is important in prospecting for, mining and
processing of minerals. In developing countries, mineral exploitation has
been affected by low levels of technology.
7) Availability of capital. A large capital outlay is required in mining.
Countries with inadequate capital rely on foreign investors who always
have an upper hand in mining over the local people.
8) Availability of market. Unstable world market prices may affect mining .
for example, Zambia where copper has been a major foreign exchange
earner was heavily affected by falling of the copper prices in the 1970s.
some mines were closed down and the country resorted to other
economic practices to avoid effect on balance of payment.

Page | 117
Prefer Calling Sir Obiero Amos @ 0706 851 439 for F1-F4 All Subjects Notes
Methods of Mining
There are basically two main types of mining:
• Open-cast or surface mining

• Underground mining

Methods of Mining

.
1. Open-cast or Surface mining.
In this type of mining the minerals or rocks that are to be mined are exposed
at the surface or very close to the surface. To extract the materials, giant
earthmovers remove the top soil and the rocks and the material is extracted.
The following are types of surface
mining processes:

a) Strip mining process


As the name suggests the surface of the earth is stripped. In this process the
soil and rocks that lie above is removed generally by heavy machinery and
then the mineral ore is extracted. This type is possible only when the targeted
materials are relatively near the surface. The mineral generally extracted is
coal or some kinds of sedimentary rocks. In Kenya Athi River and Bamburi
Limestone mining is done by stripping

Page | 118
Prefer Calling Sir Obiero Amos @ 0706 851 439 for F1-F4 All Subjects Notes
b) Alluvial/Placer mining process
In this process alluvial deposits in sand or gravel are extracted. This process
does not involve the usage of any sort of heavy materials and can be
considered relatively easy when compared to others. Generally gold and
other gemstones are mined in this process.

c) Hydraulic mining process


In this process high pressure water jets are used to dislodge rocks and
minerals. During earlier times gold was found in an easier way by this
method. However, this process has been discontinued due to environmental
concerns.

d) Dredging process
It is the process of underwater excavation by deepening a water body. In
this method sediments and other substances are removed from harbors,
rivers and other water bodies and minerals are extracted.

e) Panning.
This process involves digging out of the sand or gravel from the bed of the
river and whirling it with water in a shallow pan. The pan is tilted in such a
way that the lighter sand/gravel is washed on the side leaving the heavier
mineral. For example, mining of gold at Rongo in Migori and on river Morun
beds in west Pokot.

f) Open pit process


The easiest and the cheapest way to mine materials that are close to the
surface, large open holes are dug in the ground. Sometimes, explosives are
used to get large blocks of rocks out of the way.

Page | 119
Prefer Calling Sir Obiero Amos @ 0706 851 439 for F1-F4 All Subjects Notes
2. underground mining
There are five processes. They are:
a) hill Slope boring
A type of underground mining, slopes are made into the ground and the
desired material is accessed. This is done when the minerals are located far
enough and surface mining cannot be employed to reach it. Generally, coal
is mined in this way.

b) Drift/Adit mining process


This process is carried out when the material is situated sideways of a
mountain. The materials are easier to access and the mouth is made slightly
lower than the resource area so as to allow gravity to pull down the
materials easily. Generally, coal or iron ore is mined through this process.

c) Shaft process
The deepest form of underground mining, this is done by excavating a
vertical passageway deep down. The materials to be extracted are situated
deep inside and elevators are used to take the miners up and down. It is kept
in mind that the tunnels are made airy for miners to work without any
problems.
Generally, coal is mined in this process.

d) Solution method.
This is a method of underground mining which involves sinking of pipes
with superheated water or steam to the deposit of minerals such as salt
potash or sulphur. The mineral dissolves in water, which is then pumped
to the surface. At the surface, the water is evaporated and the mineral
extracted.

e) Drilling.
It is commonly used in the exploitation of petroleum and natural gas. The
deposit is reached by boring wells. The mineral is then brought to the surface
either under its own pursuer or by pumping.

Page | 120
Prefer Calling Sir Obiero Amos @ 0706 851 439 for F1-F4 All Subjects Notes
Significance of minerals in Kenya.
a) Promotion of industrial growth. Minerals form raw materials for
manufacturing industries. For example, the cement and lime factories in
Athi River, Bamburi, Koru and Likoni depend on limestone mining. Glass
manufacturing factory in Nairobi and Mombasa rely on soda ash mining.
Refining of crude oil at Changamwe has promoted oil related industries
like the manufacture of plastics.
b) Minerals have stimulated development of transport links. Roads and
railway lines have been built to areas previously inaccessible due to
mining.
c) Mining has promoted provision of employment opportunities. For
example, soda ash mining in Magadi employs local people.
d) Minerals have stimulated development of settlement. Urban areas have
sprung up in mining areas like Magadi, Macalder, Kakamega, and
Chepkorio in Kerio Valley.
e) Mining has led to development of social amenities. E.g Schools,
electricity, water and hospitals have been made available in mining areas.
f) Mining has led to improvement of standards of living for the people
employed in mining.
g) Minerals are a foreign exchange earner. For example, soda ash is sold to
Japan and India, fluorspar to Europe.

DISTRIBUTION OF MINERALS IN EAST AFRICA.


East Africa is not well endowed with deposits of important industrial
minerals like coal and iron ore. However, other important minerals like
diamond, copper and soda ash exist. Soda Ash is mainly mined in Kenya at
lake Magadi, copper in Uganda at kilembe mines and diamond in Shinyanga
district in Tanzania.
Map of east Africa showing mineral
distribution.

Page | 121
Prefer Calling Sir Obiero Amos @ 0706 851 439 for F1-F4 All Subjects Notes
Use the text book to mark out the Minerals
MINERAL PLACE MINED
Soap stone Soneka in Kisii
Fluorspar Kerio Valley at kimwarer
Gold Macalder in Migori
copper Macalder in Migori
Soda Ash(trona) Lake Magadi
Natural CO2 Kereita near uplands and
Esageri near Eldama
Ravine
Diatomite Kariandusi near Gilgil, at
Gicheru in Nyandarua
ranges
Salt On the surface of lake
Magadi, at the shores of
the Indian ocean at
Ngongoni near Malindi
Gypsum Elwak
Limestone Athi River near Nairobi,
Bamburi in Mombasa,
Sultan Hamud, Homa Bay
and Koru
near Kisumu
Gemstone Near Voi and Garnets at
Mwatate in Taita.
Titanium In Kwale

Problems facing the mining industry in kenya.


a) Inadequate capital. Exploration for oil in the coastal and northern regions
has never been completed due inadequate capital.
b) Insufficient skilled personnel. Mineralogists and petrologists are lacking.
This leads to overreliance on expatriates for prospecting minerals and
mining. Currently, oil prospecting and drilling in Turkana is being
undertaken by foreign investors.
c) Remoteness and poor transport systems. Areas of mineral deposits are
sometimes inaccessible thus making prospecting and mining very
difficult.

Page | 122
Prefer Calling Sir Obiero Amos @ 0706 851 439 for F1-F4 All Subjects Notes
d) Control by multinational companies. As a result much of the export revenue
is siphoned to foreign countries as salaries and dividends.
e) Small and uneconomic mineral deposits. For example Asbestos, Mica,
Copper, Beryl, Gold, Silver, Limestone and Quartz all exist at Nyiru in
Turkwel Valley but in small uneconomical quantities. They have never
been exploited.
f) Pollution of the environment. Dereliction and reckless disposal of materials
from mining are a major problem emanating from diatomite mining at
kariandusi.
g) Risks of death. This is especially when the mines collapse and bury people.
For example, gold mines in Maragoli, Kakamega and south nyanza.
h) Lack of power supply. This leads to high costs of mining and prospection.
Minerals exist in Ewaso Nyiro valley around Moyale, Ramie El Wak and
Wajir, but lack of power supply has made mining impossible.
i) Land use conflict. For example, Titanium mining in Kwale. There has been
a stand-off between TIOMIN Company and the local people.

Effects of mining on the environment.


a) Dereliction of land.
This results from the ruthless exploitation of natural resources
without considering the future
Derelict land is one that has been abandoned because it is damaged.
Examples of derelict land In Kenya are the Murram pits, stone quarries, and
limestone mining areas in Bamburi and Athi River.
Consequences of dereliction include;
~ Wastage of agricultural land
~ Ugliness of the landscape as it losers its aesthetic values.
~ Wastage of industrial and housing land if near a town
~ Causes landslides
~ Health hazard due to pools of water collected

Page | 123
Prefer Calling Sir Obiero Amos @ 0706 851 439 for F1-F4 All Subjects Notes
Derelict land can however be reclaimed or rehabilitated to become useful for
agricultural purposes and other uses. For example, the limestone mines in
Bamburi have been reclaimed to form the Bamburi Natural park with a
variety of animals.

b) Pollution of air /land noise.


The dust and toxic gases produced during the mining pollute the
atmosphere/are a health hazard. Mining can pollute streams and disrupt
water supplies. Clear water often turns reddish-orange if it contains a high
concentration of iron. Mining may also cause noise pollution.
c) Loss of biodiversity. Wildlife often suffers severely as a result of mining.
In the short term, all species are either destroyed or displaced from the
area of the mining. Some of the biodiversity may become victims of
pollution
d) Soil erosion/degradation. Mining leads to increased Erosion especially
when the protective plant cover is removed and the remaining soil is not
stabilized. Mining disturbs and may even destroy the beneficial micro-
organisms in the topsoil in addition to eliminating existing vegetation and
altering the soil profile, or the natural soil layers.
e) Mining also may degrade the productive capacity of adjacent land. Spoil
placed on adjacent land that has not been properly prepared may erode
and thereby cover topsoil or introduce toxic materials to the soil.
f) Mining also may alter the natural topography of the area in ways that
prevent a return to the previous land use, such as farming.
g) Mining activity can also affect the quantity and quality of groundwater
supplies. Blasting activity and subsidence from underground mining may
break up the impermeable layers of rock that holds water in these
aquifers, even where the overburden is not being extracted.
h) Water collects in the hollows left by open cast mines creating ponds which
become habitats for disease causing organisms.
i) Mining causes mass wasting. The tremors caused by heavy equipment
and explosives in mining may trigger off mass wasting.

Page | 124
Prefer Calling Sir Obiero Amos @ 0706 851 439 for F1-F4 All Subjects Notes
j) Mining may trigger off unemployment. Since it is a non-renewable
resource, on exhaustion, workers become jobless.
k) Mining may lead to exploitation of other countries. Countries lacking in
capital and skilled manpower may be exploited by the developed nations
to the level of gaining very little from their minerals.
l) Mining may hinder planning. Priority planning lacks in countries like
Zambia due to total overdependence on copper mining. Agricultural
sector has been heavily affected by this.
m) Political instability. In countries like DRC and Guinea , the instability
witnessed for long after independence has been majorly due to mining;
when foreign companies play Africans against fellow Africans for their
benefit.

Possible solutions to the problems due to mining.


a) Dereliction related problems can be solved through legislation that would
curb against improper disposal of waste and conditional reclamation of
scars left behind after mining.
b) Diversification of means of production can help solve the problem of
unemployment when minerals exhaust.
c) Mines should be nationalized as a way of curbing against foreign
exploitation.
d) The activities of foreign companies should be closely monitored by
governments to curb against political instability

EXPLOITATION OF MINERALS IN SELECTED COUNTRIES


MINING OF SODA ASH (TRONA) ON LAKE MAGADI IN KENYA
Location.
The deposits re located about 120km south west of Nairobi on the flour of
the rift valley. Magadi Soda Company has been in operation since 1911 when
the deposits were surveyed and the company formed.

Page | 125
Prefer Calling Sir Obiero Amos @ 0706 851 439 for F1-F4 All Subjects Notes
Occurrence of Soda ash.
Soda ash occurs in form of crystals called trona. The Trona deposits are a
solution of different sodium salts (Sodium sequicarbonate and sodium
chloride are the most common) in Lake Magadi. Soda ash is obtained from
sodium sequicarbonate.

Formation.
When rain water percolates into cracks on the floor of the rift valley, it is
heated by high underground temperatures which facilitate the dissolving of
soda salts that are found in the underground rock of the rift valley.
The dissolved soda is ejected to the surface through hot springs and then
flows to the floor of Lake Magadi. Due to high lake temperatures, it is rapidly
evaporated leaving behind the solid crystals of trona on the surface.

Extraction and processing. (Method- placer mining/dredging)


A dredger floats on the lake water from where it excavates trona crystals
from the bed. The crystals are fed in a crusher and mixed with a solution
(liquor) from the lake bed. The mixture is pumped through a pipe to the
factory, where the crystals are cleaned. The liquor is removed back to the
lake. After cleaning, the crystals are heated and converted into soda ash. The
soda ash is then reduced into required sizes and packed for export.
Uses.
a) Soda ash is used to manufacture glass, soap, caustic soda and
detergents.
b) NaCo3 is also used in other industries like paper making, oil
refining and textile industries. c) It is used for softening water.

Marketing.
It is consumed locally mostly. Part of it is exported to
Japan and India.

Page | 126
Prefer Calling Sir Obiero Amos @ 0706 851 439 for F1-F4 All Subjects Notes
Benefits of soda ash mining to the economy.
a) It earns the country foreign exchange

b) It generate jobs to Kenyans

c) Leads to uplifting of living standards

d) Development of settlements e.g. Magadi town.

e) Development of transport system

f) It has led to development of social amenities.

Problems facing soda ash mining in Magadi.


a) The country is facing stiff competition from developed countries with
large soda deposits such as
USA and Israel
b) The low value of salt (Sodium chloride) is insufficient to meet its
production costs qualified.
c) High labour costs since workers have to be given high incentives to agree
to work in the harsh hostile environment of Lake Magadi

GOLD MINING IN SOUTH AFRICA


Occurrence.
Gold is shinny and yellow in colour. It is soft and hardens only when it is
mixed with other metals. It is does not rust and cannot deteriorate. It appears
in grains in solid rock. Gold is found in a rock called a banket. This ore is
found in quartz veins, metamorphic rocks or in redistributed sedimentary
rocks.

Location.
South Africa is the leading producer of Gold in the world. It contributes 70%
of the world’s output.
The Witwatersrand Gold fields near Johannesburg were discovered in 1884
and mining started in 1886. Gold occurs in reefs or lodes.

Page | 127
Prefer Calling Sir Obiero Amos @ 0706 851 439 for F1-F4 All Subjects Notes
SOUTH AFRICAN GOLD-FIELDS

Methods of extraction
The Gold bearing reef is reached by sinking shafts as deep as 6000m.
Horizontal shafts are then run to the bankets at different levels from the
main vertical shaft. The banket is removed from the reef and hauled up to
the surface.
At the surface, the banket is crushed into chips, and then mixed with water
before being ground into fine dust/pulp to recover gold.
Alluvial gold is recovered through panning.

Processing and marketing.


94% of Gold is removed from the pulp by solution in Cyanide to produce
potassium Gold Cyanide. The solution contains some amount of Uranium
and thus it is not pure Gold. It is then mixed with Zinc Dust which causes
the gold to precipitate out leaving uranium. It is then fed into sulphuric acid
tanks to dissolve the uranium. Gold is then melted out and formed into bars.
3 tonnes of extracted banket can produce 30 grammes of gold.
Only 2.3 tonnes of gold are consumed locally in south Africa in a year. Gold
is used in making of jewellery and ornaments, in chemical industries, in
dentistry and for medals.

Page | 128
Prefer Calling Sir Obiero Amos @ 0706 851 439 for F1-F4 All Subjects Notes
Contributions of gold to the economy of South Africa.
a) It earns the country foreign exchange which is important in paying
international debts.
b) It is a source of employment to people thus raising their living standards.
c) It has contributed to urbanization. E.g the formation of the Witwatersrand
conurbation is due to gold mining.
d) It has stimulated growth of other industries by providing market for
secondary industries through linkages and external economies of scale.
E.g engineering, footwear, electrical and construction industries.
e) It has stimulated growth of modern infrastructure and amenities. E.g
shopping centres, entertainment, industrial and residential zones.

Problems facing gold mining.


a) Increased costs of mining gold as the mines continue to get deeper.

b) High cost of labour due to increased demand of higher wages and social
amenities for workers. Other sectors of the economy are competing for
the same labour thus posing difficulties in gold mining in future.
c) Inadequate water. 2 tonnes of water are needed for one tonne of ore. The
area receives seasonal rainfall and its population needs water.
d) Great depth of the mines leading to “pressure bursts” underground
causing ground shattering as the ore is removed.
e) Cooling and ventilation in such depths is a problem, The problem of
pumping out saline water. The costs for overcoming this are enormous.
f) Deteriorating quality. The Gold Quality is getting poorer and poorer than
it was some years back.
g) Exhaustion of some mines since gold is non-renewable.

Page | 129
Prefer Calling Sir Obiero Amos @ 0706 851 439 for F1-F4 All Subjects Notes
DIAMOND IN SOUTH AFRICA.
Diamonds are the hardest rocks known to man and are made of
carbon.
Diamonds are associated with volcanic activities. They occur in igneous
rocks in pipes. They may form at great depth and are carried up the pipe
during eruption.
Diamonds occurring in pipes exposed to the surface are obtained by open-
cast mining and are known as gem diamonds. (Also see alluvial diamonds
formed after erosion of exposed volcanic pipes and deposited in sea by rivers)

Mining, processing and marketing of diamond.


The ore is blasted, lifted to the surface, crushed into small pieces and
washed. It is then passed through filtering screen and then through a special
solution. (The diamond plus other rock remains in water are passed over a
greased rotating table to repel water. Since diamond does not get wet, it
sticks on the table while the wet rock pieces are repelled off.) Uses.
a) Diamonds are used in the jewellery industry, for polishing, for drilling,
and for cutting instruments.

Contribution of diamond to the economy.


a) Diamonds; earn foreign exchange,

b) have led to development of towns,

c) have created employment

d) Contributed to development of infrastructure. The problems


facing diamond mining a) exhaustion,
b) unstable world market prices,

c) high costs of processing

d) Inadequate labour.

Page | 130
Prefer Calling Sir Obiero Amos @ 0706 851 439 for F1-F4 All Subjects Notes
PETROLEUM IN THE MIDDLE EAST.
The Middle East countries of Saudi Arabia, Irag, Iran, Kuwait and UAE
provide 33% of the world’s total output of petroleum. Other producers are
Bahrain, Yemen, Oman, Syria and Qatar.
The drilling and exploration of the crude oil is done by foreign companies
from Europe and USA. Refined oil is then transported by pipeline from the
refineries on the Persian gulf to the Mediterranean ports of Haifa, Beniyes
and Saida. 35% of the crude oil is re4fined locally. The middle east countries
are the most important members of OPEC.
OILFIELDS IN THE MIDDLE EAST

Lebanon

Beniyas

Mediterranean Syria
sea

Haifa Iraq

Iran
Jordan

Qatar
Egypt
Kuwait

KEY Persian Gulf


Main OilFields
Saudi Arabia
U.A.E
Refining centres

OIL IN SAUDI ARABIA.


Location.
Exploration started in 1933 by the Standard oil company of California.
Drilling began in 1938. The largest oilfields I Saudi is the Dammam Oilfields.
Other oil field are Ghawar 9which has become the world largest oilfield after

Page | 131
Prefer Calling Sir Obiero Amos @ 0706 851 439 for F1-F4 All Subjects Notes
discovery in 1948), Abgaig, Aindar and the offshore wrecks of safaniya and
Dhahran. Saudi Arabia crude oil is piped to the Persian Gulf for refining.
The country had the largest oil reserves in the world and joined OPEC in
1960.
Currently, ARAMCO is the company responsible for the
exploitation of most of its oil.

OIL IN IRAG.
Oil was discovered here in 1927. The major oilfields are around Kirkuk and
Mosul in the northern part. The oil is marketed by pipeline to the
Mediterranean ports in Libya. Syria and Lebanon for refining and export.
Drilling is mostly done by the British companies.
Oil in Iran.
Oil was first drilled in Iran in 1913 by the British Anglo-Persian Oil
Company. In 1951, the first National Oil Company was formed . Major Oil
fields are in SW near Bahnegan, Maijidi-Sulaiman, Lali, Nafti-IShah and
Agha Jari.
Petroleum accounts for 90% of Iran’s exports and is the 2 nd largest producer
after Saudi Arabia.
Export are done by the Anglo-Iranian Oil Company who are the forerunners
of British Petroleum (BP).
Oil in Kuwait.
One of the world’s wealthiest countries due to the oil whose mining began
in 1946. Mining is done by the Kuwait Oil Company (a joint American and
British Company) the oil reserves are about 94 Billion Barrels- 10% of the
world’s reserves.. Petroleum accounts for nearly 90% of the export revenue.
2 billion oil Barrels were torched in 1991 by the Irag soldiers.

Page | 132
Prefer Calling Sir Obiero Amos @ 0706 851 439 for F1-F4 All Subjects Notes
UAE.
The independent states along the Persian GULF E.G Abu Dhabi, Dubai and
Sharjah. Its oil was discovered in 1954 and brought wealth to the region
initially one of the least developed in the world.
By 1970 UAE had one of the world incomes. Oil reserves account for 9.4% of
the world’s reserves. National Oil Company was formed in 1975.
OTHER PRODUCERS.
Bahrain, Qatar and Yemen

Page | 133
Prefer Calling Sir Obiero Amos @ 0706 851 439 for F1-F4 All Subjects Notes
MODEL KCSE PAPERS COVERING FORM ONE WORK
CHAPTER 1
INTRODUCTION TO GEOGRAPHY
1. What is practical geography? (2mks)
2. Name two branches of geography (2mks)
3. Name any three study areas in physical geography (3mks)
4. Define the term environment (2mks)
5. Explain three reasons for importance of studying geography
(3mks)
6. Name five human features (3mks)
7. What is habitat (2mks)
8. List six disciplines related to geography (6mks)
9. Differentiate between each of the following:
▪ Demography and population geography
▪ Economics and economic geography (4mks)
10. Name two Greek words from which term geography originate
(2mks)

CHAPTER 2
THE EARTH AND THE SOLAR SYSTEM
1. (a) State two effects of the rotation of the earth
(2mks)
(b) Study the diagram below and answer the questions that follow

Page | 134
Prefer Calling Sir Obiero Amos @ 0706 851 439 for F1-F4 All Subjects Notes
(i) Which movement of the earth is represented by the diagram?
(1mk)
(ii) Give two effects of the movement represented by the diagram
(2mks)
2. The diagram below represents the structure of the earth. Use it to answer
question

(a) Name
(i) The parts marked P and Q
(2mks)
(ii) The discontinuity marked R
(1mk)
(b) State three characteristics of the mantle (3mks)
3. The diagram below shows the composition of the solar system

Page | 135
Prefer Calling Sir Obiero Amos @ 0706 851 439 for F1-F4 All Subjects Notes
Name the planets marked F and G (2mks)
4. a) What is the solar system? (1mk)
b)Use the diagram below to answer the questions that follow.

i) What type of eclipse is represented by the diagram? (1mk)


ii) Name the features marked L and M (2mks)

5. (a) (i) Give the two dates in a year during which the number of hours of
darkness is equal in both the north and south poles. (2mks)
(ii) Why do the lengths of days and nights vary from one part of the
earth to another? (2mks) (b) The diagram below shows the revolution of
the earth around the sun. Use it to answer the questions that follow

(i) If the earth takes 366 days to make a complete revolution during a leap
year, how long will it take to move from position 1 to position 4? (2mks)
(ii) What season is experienced in the southern hemisphere when the earth is
in Position 1? (2mks)

Page | 136
Prefer Calling Sir Obiero Amos @ 0706 851 439 for F1-F4 All Subjects Notes
6. Define the following,
i. Solar system
ii. Galaxy iii. Star
iv. Asteroids (6mks)
7. Differentiate between the following

(a) Latitude and longitude


(b) Dateline and international dateline
(c) Meteors and Meteorite. (6mks)
8. State three differences between solar eclipse and lunar eclipse.
(2mks)
9. State four factors that support life on planet earth.
(4mks)
10. (a) List four effects of earth rotation.
(4mks)
(b) At Nairobi on longitude 37°E local time is 1 p.m. What time would it be
at Garissa on longitude 41 °E?
(4mks)
11. (a) Define equinox.
(2mks)
(b)State characteristics of summer solstice.
(4mks)
12. Fill in the table from (a) - (f) (10mks)
Property Major Thickness Density Temperature
s/Layer constituent
Outer crust (a) iii. 16-24 (b)
kms
Inner crust ii. S (c) 2.8-30
Magnesium gms/cc
Asthenosphere i. Iron 2900 kms (d) 5000 C
Centrosphere ii. Nickel (e) (f)

Page | 137
Prefer Calling Sir Obiero Amos @ 0706 851 439 for F1-F4 All Subjects Notes
13. State three weaknesses of the passing star theory.
(6mks)
14. Differentiate between hydrosphere and atmosphere.
(4mks)
15. Explain reasons for flattening and bulging of earth.
(4mks)

CHAPTER 3
WEATHER
1. (a) How does a sea breeze occur?
(2 mks) (b) Use the map of Africa below to answer questions (b) (i)

(i) Name the ocean currents marked H, J, and K


(3 mks) (ii) State two effects of a warm ocean current on the adjacent
coastlands (2 mks) 2. (a) State two conditions that are
necessary for the formation of fog. (2 mks)
(b)The diagram below shows some types of clouds. Use it to answer the
questions that follow.

Page | 138
Prefer Calling Sir Obiero Amos @ 0706 851 439 for F1-F4 All Subjects Notes
(i) Name the clouds marked R
(1 mk)
(ii) Give two weather conditions associated with cumulonimbus clouds.
(2 mks)

3. (a)the tables below represent rainfall and temperature of stations X and


Y.Use them to answer questions (a) and (b)
MONTHS J F M A M J J A S O N D
TEMPERATURE IN 0c 30 31 31 31 30 2 2 28 28 29 29 30
9 9
RAINFALL IN MM 250 250 32 30 213 2 2 25 10 27 38 20
5 0 5 5 0 5 0 0

MONTHS J F M A M J J A S O N D
TEMPERATURE IN 0C 21 20 20 17 15 13 12 13 15 16 18 20

RAINFALL IN MM 12 12 15 50 90 110 87 87 50 35 20 15
a) (i) For each of the two stations calculate the mean annual
temperature.
X -
Y -
(ii) Calculate the annual rainfall for station Y

(iii) On the graph paper provided, draw a bar graph to represent rainfall
for station x. Use vertical scale of 1cm to represent 50mm
b) Describe the climatic characteristics of station Y.

4. (a) Describe a suitable site where you would locate a weather station in
your School (2 mks) (b) Give reasons why a Stevenson’s screen is:
(i) Painted White (2 mks)
(ii) Has louvers (2 mks)
5. Define relative humidity. (2 mks)
6. (a) Identify four characteristics of convectional rainfall. (4mks)
(b) State the difference between radiation fog and advection fog.
(4mks)

Page | 139
Prefer Calling Sir Obiero Amos @ 0706 851 439 for F1-F4 All Subjects Notes
7. (a) Briefly describe how the six thermometers operate. (5mks)
(b) Three ways in which clouds are classified. (3mks)
8. (a) Give three precautions to be taken when citing a weather station.
(3mks)
(b) State three factors determining the amount of solar radiation reaching
the earth's surface. (3mks)

9. Define the following terms: (i) Climate


(ii) Weather forecasting
(iii) Absolute humidity
(iv) Weather lore (5mks)
10. State the advantages of studying weather through field work.
(5mks)
11. (a) Describe how you would use the following apparatus during a field
study.
Rainfall gauge, maximum and minimum thermometers.
(3mks)
(b) Identify and explain the formation of the type of rainfall found in the Lake
Region or Kenya. (8mks)
(c)Briefly write down two problems associated with the type rainfall above.
(4mks)
12. (a)List four problems of weather forecasting.
(4mks)
(b) State four ways in which weather forecasting is important to the
human activities. (4mks)

Page | 140
Prefer Calling Sir Obiero Amos @ 0706 851 439 for F1-F4 All Subjects Notes
CHAPTER 4
STATISTICAL METHODS
1. (a) Define the following terms
- Statistics
- Statistical data
- Statistical methods
(6mks)
(b) State two types of statistical data.
(2mks)
(c) Write down two types of questionnaires.
(2mks)
2. (a) What factors must be considered in selecting methods of data
collection. (3mks)
(b) Differentiate between discrete data and continuous data giving relevant
examples. (4mks)
3. (a) What is sampling
(1mk)
(b) State 3 types of sampling.
(3mks)
4. (a) Name two main methods used in analyzing statistical data.
(2mks)
(b) What is the significance of statistics in geography?
(5mks)

Page | 141
Prefer Calling Sir Obiero Amos @ 0706 851 439 for F1-F4 All Subjects Notes
CHAPTER 5
FIELD WORK
PAST KCSE QUESTIONS ON THE TOPIC
1. State two ways in which information collected during the field study
would be useful to the local
community. 2mks
2. Your class is required to carry out a field study of a river. What would
be the advantage of dividing the class into groups according to the
stages of the long profile 3 of a river?
3. What would be the disadvantages of c using secondary data in this
kind of a field study?
4. You intend to carry out field study on population in the local open air
market,
(i) State three reasons why it would be necessary for you to visit the
market before actual field study. (ii) Give two methods you would
use to collect information on pollution. , (iii) State three follow up
activities necessary for the study.
5. You are supposed to carry out a field study on the uses of vegetation
in the area around your school.
(a) Give four uses of vegetation you are likely to identify during the
study. (3mks) (b) Why is it necessary to sample part of the
forest for the study?
6. List three types of fieldwork. (3mks)
7. Explain the importance of field work.
(5mks)

Page | 142
Prefer Calling Sir Obiero Amos @ 0706 851 439 for F1-F4 All Subjects Notes
CHAPTER 6
MAP WORK
Study the map of Kisumu East (1:50,000) and answer the following questions.
(a) (i)What is the bearing of the Trigonometrical station at grid reference
081980 from the rock outcrop at
grid reference 071992?
(2mks)
(ii) Measure the length of the all weather road (bound surface) 1321,
from, the junction at grid reference 974911 to the edge of the map, grid
reference 947967. (2mks) (b) (i) Describe the relief
of the area covered by the map.
(ii)Explain how relief has influenced the settlement in the area
covered by the map. (8mks) (c) Citing evidence give three economic
activities carried out in the area covered by the map.
(d) Students from the school at Masago (grid square 0681) carried out field
study of the course of river Ombeyi.
(i) State three findings they are likely to have come up with.
(3mks)
(ii) Give three advantages of studying rivers through field
work

Page | 143
Prefer Calling Sir Obiero Amos @ 0706 851 439 for F1-F4 All Subjects Notes
CHAPTER 7
ROCKS AND MINERALS
1. (a) Describe the following characteristics of minerals
(i) Colour
(2mks)
(ii) Cleavage
(2mks)
(iii) Hardness
(2mks)
(b) Give two types of igneous rocks
(2mks)
(c) State four uses of rocks
(4mks)
2. (a)State two characteristics of sedimentary rocks
(2mks)
(b) Give two examples of chemically formed sedimentary rocks (2mks)

3. (a)Name the type of rocks which results from the metamorphism of:
(i) Granite
(ii) Clay (2mks)
(b) Give two reasons why sedimentary rocks are widespread in the
coastal plain of Kenya. (2mks)
4. (a)(i) What is a rock? (2mks)
(ii) Describe three ways through which sedimentary rocks
are formed
- Mechanically formed

- Organically formed

- Chemically formed (6mks)


(b)Describe two process through which sedimentary rocks
changer into metamorphic rocks 5. (a) Differentiate between plutonic
rocks and volcanic rocks.

Page | 144
Prefer Calling Sir Obiero Amos @ 0706 851 439 for F1-F4 All Subjects Notes
CHAPTER 8
MINING
1. State three conditions that are necessary for the formation of
petroleum (3mks)
2. (a) Use the map of Africa to answer question (a) (i)

i) Name the minerals mined in the areas marked S, T and V.


ii) State two formations in which mineral ores occur.

b) Explain four problems, which Zambia experiences in the


exportation of copper.
c) Explain three ways in which coal contributes to the economy of
Zimbabwe.
d) Describe three negative effects of open cast mining on the
environment.

Page | 145
Prefer Calling Sir Obiero Amos @ 0706 851 439 for F1-F4 All Subjects Notes
ANSWERS TO MODEL KCSE PAPERS COVERING FORM ONE WORK
CHAPTER 1
INTRODUCTION TO GEOGRAPHY
1. What is practical geography? (2mks)
~ Collective term for methodologies of fieldwork, maps and map work and
photograph interpretation
used in study of geography.

2. Name two branches of geography (2mks)


~ Physical geography

~ Human geography.

3. Name any three study areas in physical (3mks)


geography
~ Climate

~ Rocks and minerals

~ Earth and the solar system

~ Soil

4. Define the term environment (2mks)


~ External conditions surrounding of an organism.
5. Explain three reasons for importance of studying geography
(3mks) ~ Provides knowledge that promote
conservation of resources ~ Admission in careers that
generate income e.g. Geologist.
~ Learn skills in time management useful in personal activities
6. Name five human features (3mks)
~ Transport lines e.g. roads.
~ Settlement structures e.g. houses
~ Drainage features e.g. boreholes, water dams
~ Industries e.g. coffee mills

Page | 146
Prefer Calling Sir Obiero Amos @ 0706 851 439 for F1-F4 All Subjects Notes
~ Farms e.g. tea estates.
7. What is habitat (2mks)
~ Part of physical conditions that provide home in which certain
organisms live.
8. List six disciplines related to geography
(6mks)
~ History, Biology meteorology, Demography, Sociology, Agriculture,
Economics
~ Physics
~ Chemistry
~ Geology
~ Medicine
9. Differentiate between each of the following:
▪ Democracy and population geography

▪ Economics and economic geography (4mks)


~ Demography is the study of human population dealing with numerical
aspects of population while population geography is branch of human
geography dealing with population explaining where and why people
live.
~ Economics deals with availability of resources while economic
geography is branch of geography that deals with location and
distribution of resources.
10. Name two Greek words from which term geography originate
(2mks) ~ Geo, graphein

Page | 147
Prefer Calling Sir Obiero Amos @ 0706 851 439 for F1-F4 All Subjects Notes
CHAPTER 2
THE EARTH AND THE SOLAR SYSTEM
6. (a) State two effects of the rotation of the earth

~ Causes day and night/apparent movement of sun


from east to west.
~ Causes differences in time at different longitudes.
~ Causes deflection of winds\ ocean currents.
~ Causes rising and falling of sea tides.
~ Causes variation in atmosphere pressure on the
earth surface.
(b) (i) which movement of the earth is represented
by the diagram? ~ Earth revolution
(ii)Give two effects of the movement represented by the diagram
~ Causes changes in position of midday sun at different times of the year.
~ Causes varying length of the days and nights in northern and southern
hemisphere.
~ Causes changes in seasons; spring, summer, autumn and winter.
~ Causes lunar eclipse
7. (a)(i) Name the parts marked P and Q
P - Atmosphere
Q - Barysphere/Centrosphere/core
(ii) The discontinuity marked R
R -Mohorovicic discontinuity/Moho
discontinuity (b) State three
characteristics of the mantle
~ Divided into two-upper and lower mantle
~ Mantles' main constituent minerals are Ferro-magnesium and silicate.
~ Mantle is about 2, 900 km thick.
~ Upper mantle has low temperature than lower mantle.

Page | 148
Prefer Calling Sir Obiero Amos @ 0706 851 439 for F1-F4 All Subjects Notes
~ Mantle has temperatures of about 1000°C.
~ Mantle is made up heavier rocks than rocks of earth crust.
~ Upper mantle is made up of an elastic solid/semi-molten
~ Inner mantle is made up an elastic solid/semi molten basic rocks/
viscous liquid.
8. Name the planets marked F and G
~ Mars
~ Neptune
9. (a) What is the solar system?
~ Solar system refers to the composition of the sun, the planets and other
heavenly bodies related to the
sun.
(b) (i) What type of eclipse is represented by the diagram?
(1mk)
~ Solar eclipse
(ii) Name the features marked L and M
(2mks)
L - Moon
M Shadow

10. (a) (i) Give the two dates in a year during which the number of hours of darkness
is equal in both the north and
south poles. (2mks)
~ 21st March and 23rd September
(ii) Why do the lengths of days and nights vary from one part of the
earth to another? (2mks) ~ Due to revolution of the earth.
(b) (i) If the earth takes 366 days to make a complete revolution during a leap year,
how long will it take to move
from position 1 to position 4? (2mks)
274.5 days

Page | 149
Prefer Calling Sir Obiero Amos @ 0706 851 439 for F1-F4 All Subjects Notes
(ii) What season is experienced in the southern hemisphere when the earth is in
Position 1? (2mks) Summer season
11. Define the following,

(i) Solar system - Organization made up of the sun with the nine
planets orbiting around it and heavenly bodies,
(ii) Galaxy - Group/cluster of stars in the universe.

(iii) Star - Hot mass of glowing gases that transmit light to outer bodies.

(iv) Asteroid - Small planet-like objects orbiting around the sun


between the planets of Mars and Jupiter.
12. Differentiate between the following

(a) Latitude and longitude

~ Latitude is the distance north or south of equator measured as an angle


from the earth's centre while longitude is the distance of the earth's
surface measured east or west of prime meridian and expressed as an
angle.
~ Latitude is imaginary line running from East to West showing how far
north or South a place is from Equator.
(b) Dateline and international dateline

~ Dateline is line 180° at which a day is lost or gained while international


dateline is zigzag line along longitude 180° deviating land surfaces and
at which day is lost or gained.
(c) Meteors and Meteorite.

~ Glowing objects that quickly cross the sky before they burn up and
disappear while meteorites are those meteors that pass through the
atmosphere brightly but do not burn up.
13. State three differences between solar eclipse and lunar eclipse.
(2mks)
~ In solar eclipse moon lie between sun and earth while in lunar eclipse
earth lie between moon and sun.
~ In solar eclipse shadow of moon is cast on earth while in lunar eclipse
shadow of earth is cast on moon.

Page | 150
Prefer Calling Sir Obiero Amos @ 0706 851 439 for F1-F4 All Subjects Notes
~ Solar eclipse occurs during the day while lunar eclipse occurs during
the night.
~ Lunar eclipse is caused by earth's revolution while solar eclipse is
caused by revolution of moon.
14. State four factors that support life on planet earth. (4mks) ~
Presence of water that support life.
~ Presence of atmosphere with adequate O2 and CO2 levels that support
life of animals and plants respectively.
~ Enough heat and light due to earth's favourable distance from the sun.
~ Proportional gravitational force that allow objects to be upright on the
earth's surface.
15. (a) List four effects of earth rotation.
(4mks)
~ Causes deflection of the winds.
~ Causes time difference
between Meridians
~ Causes variation in speed of
air masses.
~ Causes rising and falling of
ocean currents.
~ Causes variation in
atmospheric pressure.
b) At Nairobi on longitude 37°E local time is 1 p.m. What time would it be at Garissa
on longitude 41 °E? (4mks)
G.M East
34°E
41°E 1
p.m. ?
1° = 4 minutes
4 x 4 = 16 minutes
Local time = 1.16 p.m.

Page | 151
Prefer Calling Sir Obiero Amos @ 0706 851 439 for F1-F4 All Subjects Notes
16.(a) Define equinox.
(2mks)
~ Periods 21st March and 23rd September when the sun is overhead at
midday along the equator.
(b)State characteristics of summer solstice.
(4mks)
~ Sun is overhead at mid-day along the tropic of
cancer/Capricorn.
~ The Arctic Circle experiences 24 hrs of daylight.
~ Days are longer than nights.
~ Temperatures are high in the region
experiencing summer solstice.
~ 24 hour sunshine within the circles.
17. Fill in the table from (a) - (f)
(10mks)
(a) Silica, aluminium

(b) 2.7 gms/cc

(c) 6 -10 kms

(d) 3.0-3.3 gms/cc

(e) 3470 kms

(f) 5,500 cc

18. State three weaknesses of the passing star theory.


(6mks)
~ Chances of another star approaching the sun are minimal.
~ High temperature materials drawn from the sun would disperse rather
than condense.
~ It does not explain where the sun and the star came from.
19. Differentiate between hydrosphere and atmosphere.
(4mks)

Page | 152
Prefer Calling Sir Obiero Amos @ 0706 851 439 for F1-F4 All Subjects Notes
~ Hydrosphere is part of the earth surface covered by water masses e.g.
oceans, seas, rivers and swamps while atmosphere refers to thin layer of
gases surrounding the earth and held by earth's gravitational pull.
20. Explain reasons for flattening and bulging of earth.
(4mks)
~ Earth rotates on its own axis to make a complete turn; and its poles rotate
of this axis and pulled towards each other (centripetal forces)
~ Equator covers a long distance and therefore rotates faster, with more
speed causing a flinging force (centrifugal force).

Page | 153
Prefer Calling Sir Obiero Amos @ 0706 851 439 for F1-F4 All Subjects Notes
CHAPTER 3
WEATHER
1. (a) How does a sea breeze occur?
(2 mks)
~ During the day the land heats faster than the sea.
~ The air over the land rises
~ Cooler air from the sea blows towards the land to replace the
rising air
~ The cool air from the sea is called sea breeze
a. (i) Name the ocean currents marked H, J, and K
(3 mks)
H- Mozambique, J – Benguela, K- canary current
(ii) State two effects of a warm ocean current on the adjacent coastlands
(2 mks)
~ Raising temperature causes rainfall
2. (a) State two conditions that are necessary for the formation of fog. (2 mks)
~ Air must have abundant moisture.
~ A cloudless night to facilitate terrestrial radiation.
~ Air should be calm to remain in contact with the ground in order to be
cooled.
~ There should be gentle air currents to hold water droplets in
suspension. ~ The air must be cooled below dew point.
(b) Name the clouds marked R
(1 mk) R – Cumulus
(ii) Give two weather conditions associated with cumulonimbus clouds. (2
mks)
~ Heavy rainfall
~ Thunder and lightening
~ Dense cloud cover
~ High temperatures before rain starts.
~ Sometimes hailstones.

Page | 154
Prefer Calling Sir Obiero Amos @ 0706 851 439 for F1-F4 All Subjects Notes
3. (a) (i) For each of the two stations calculate the mean annual temperature.
X - 3°C

Y - 9°C

(ii) Calculate the annual rainfall for station Y


583 mm
b) Describe the
climatic characteristics of station Y.

4. a) Describe a suitable site where you would locate a weather station in your
School ~ Open area free of shade by trees and buildings.
~ Gentle land free of flooding
~ Area with wide view of surroundings.
~ Away from concrete surfaces.
b) Reasons why Stevenson screen is;
Painted white - can reflect direct heat from the sun.
Louvred on sides - To allow free flow of air and regulate temperature. (2 mks)
5. Define relative humidity. (2 mks)
~ Relative humidity refers to the ratio between water vapour actually
present in the air and its capacity to hold water vapour at a given
temperature.
6. (a) Identify four characteristics of convectional rainfall.
(4mks) ~ It's heavy and torrential/falls in large drops.
~ Usually accompanied by lightning and thunderstorms
~ Falls mainly in the late afternoon
~ It's highly localized and lasts for a short while (15-20 mins)
(b) State the difference between radiation fog and advection fog.
(4mks)

Page | 155
Prefer Calling Sir Obiero Amos @ 0706 851 439 for F1-F4 All Subjects Notes
~ Radiation fog forms when air in contact with the ground is cooled
through terrestrial radiation while advection fog forms when warm
moist air is cooled as it passes over cool surface e.g. land/sea.
7. (a) Briefly describe how the six thermometers operate.
(5mks)
~ When the temperature rises, the alcohol in the left hand column
expands and pushed the mercury column. The mercury in turn pushes
the mercury in the right hand column and steel metal index up.
~ The maximum temperature is shown by the end of the index pushed by
the mercury.
~ When the temperature falls, alcohol in the left hand column contracts
and pulls the index along the
tube. When the temperature rises, the alcohol expands leaving behind the
index. Then the minimum temperature is read.
(b) Three ways in which clouds are classified.
(3mks) ~ According to the altitude of their
bases.
~ Their
appearance/structure ~
Their formation
8. State three factors determining the amount of solar radiation
reaching the earth's surface. (3mks)
~ Intensity of the sun's radiation in space the average distance
from the sun.
~ The transparency of the atmosphere

~ Position of the earth in its orbit

~ The area and nature of the surface on which the rays fall.

9. Define the following.

(i) Climate

Page | 156
Prefer Calling Sir Obiero Amos @ 0706 851 439 for F1-F4 All Subjects Notes
~ It's the average weather condition of a given place over a period or time
usually (30-35 years).
(ii) Weather forecasting

~ It’s the prediction of the weather situation for a given place within a given
period of time e.g. hour, a day, a week.
(iii) Absolute humidity
~ It is the total amount of water vapour that a given volume of air can -
hold.
(iv) Weather lore

~ Refers to a body of traditional facts and beliefs relating to weather e.g. a


halo around the moon, croaking of frogs, a rainbow, and migration of
birds
10. State the advantages of studying weather through field work.
(5mks)
~ The students are able to relate what they have learnt in class to the real
environment hence making geography real and interesting.
~ It breaks the class monotony.
~ It enables learners to develop skills or observation measurement,
recording and analyzing data.
~ It improves the visual memory through observation.
11. (a) Describe how you would use the following apparatus during a
field study. Rain gauge
~ The rain gauge is kept in an open space in the weather station from
above. Its raised to avoid splashes from entering into the gauge.
~ The water collected is emptied into the measuring cylinder every 24hrs.
~ Take readings on the measuring cylinder.
~ This cylinder is graduated in mm and the level the water emptied
reaches gives us the reading amount of rainfall for the day.
~ Record the readings and interpret.
A maximum and minimum thermometer

Page | 157
Prefer Calling Sir Obiero Amos @ 0706 851 439 for F1-F4 All Subjects Notes
~ When the temperature rises, alcohol in the left hand column expands
and pushes the mercury
column and maximum temperature is read.
~ When the temperature rises, alcohol in the left hand column contracts and
pulls the index along the tube and the minimum temperature is read from
the upper end of the index.
~ After recording the reading, the thermometer is reset using a magnet.
~ Interpret the readings.
(b) Identify and explain the formation of the type of rainfall found in the Lake Region
or Kenya. (8mks)
Convectional
rainfall Its
formation
~ The intense heating from the sun results into warm air rising in form of
convectional currents.
~ The rising air reaches the high atmosphere and moisture in it condenses.
Forms clouds and falls rain.
~ It falls in the late afternoon accompanied lighting and thunderstorms.
(c) Problems associated with the type of rainfall above.

~ Lightning and thunderstorms which are destructive to life and property.


~ The torrential/large drops which are harmful to the crops and other
vegetation.
~ The hailstones also are destructive to the crop leaves.
12. (a) Problems of weather forecasting
~ Inaccurate data
~ Defective instruments
~ Personnel with limited skills
~ Vagaries of nature such as earthquakes
(b) Four ways in which weather forecasting is important to the human activities.
~ Determines times for sea and air travel.

Page | 158
Prefer Calling Sir Obiero Amos @ 0706 851 439 for F1-F4 All Subjects Notes
~ Determine time when sporting
activities take place.
~ Determines the fishing activities and
habits in the area.
~ Help determine suitable clothing for
the day.
~ Help plan farmers calendar of
activities.
~ Help plan suitable housing.

Page | 159
Prefer Calling Sir Obiero Amos @ 0706 851 439 for F1-F4 All Subjects Notes
CHAPTER 4
STATISTICAL METHODS
1. (a)
Statistics
It refers to the art or science that is concerned with the
interpretation of numeric information.
Statistical data
Refers to the information collected and arranged in a systematic
manner.
Statistical methods
Refers to the techniques used in collecting, recording, analyzing
and presenting data. (b) List two types of statistical data.
Primary data and Secondary data
(c) Write down two types of questionnaires. (2mks)
~ Closed-ended (rigid)
~ Open-ended
2. (a) What factors must be considered in selecting methods of
data collection. (3mks)
~ The method should be inexpensive.
~ Should be time saving
~ Should give accurate data.
~ Most applicable method
(b) Differentiate between discrete data and continuous data giving
relevant examples. (4mks) Discreet data refers to the non-
continuous data over time given in whole numbers only e.g.
~ Total population in a nation.
~ Monthly rainfall totals.
~ No. Of livestock per district
~ Continuous data can be given in any value including
decimals e.g. 1.8km. 3. (a) What is sampling
(1mk)

Page | 160
Prefer Calling Sir Obiero Amos @ 0706 851 439 for F1-F4 All Subjects Notes
Sampling refers to the process by which a representative portion of the
whole phenomena under study is analyzed and generalized/ generalization
is made.
(e) Types of sampling
~ Systematic sampling
~ Stratified sampling
~ Random sampling
4. (a) Name two main methods used in (2mks)
analyzing statistical data. ~ Calculation of
percentages
~ Measuring of Central tendency (mean,
median and mode) ~ Frequency
distribution
(b) What is the significance of statistics in (5mks)
geography?
~ Predicting for future trends.
~ Showing changes through time
~ Establishing Geographical relationships
~ For economic planning
~ For explaining geographical phenomena.
~ Useful for making comparisons.

Page | 161
Prefer Calling Sir Obiero Amos @ 0706 851 439 for F1-F4 All Subjects Notes
CHAPTER 5
FIELD WORK
1. State two ways in which information collected during the field study would be
useful to the local community.
~ The information on rainfall can be used by farmers to plan their
calendar of activities.
~ The information on humidity can be used in improving storage of
produce e.g. Cereals.
~ The findings can be used to plan suitable time for drying farm
produce.
2. Advantages of dividing class into groups.
~ The class will be able to study the entire course of the river.
~ Would enable them to obtain information on each stage of the river.
~ Would save on time.
~ Would enable studies to be carried out in an orderly way.
~ Would encourage participation of all members of class/ entourage
individual \ roles.
~ Would facilitate more interaction among the group members.
3. Disadvantages of using secondary data.
~ Recorded data could be out of date.
~ Condition under which data was collected may have changed.
~ Obtaining records on the particular river may be difficult.
4. (i) State three reasons why it would be necessary for you to visit the market
before actual field study.
~ To design appropriate research method.
~ To prepare the working schedule.
~ To be able to identify relevant equipment for data collection.
~ To identify suitable areas for study/ to familiarize with people who will
provide information
~ To seek permission from owners of the land.

Page | 162
Prefer Calling Sir Obiero Amos @ 0706 851 439 for F1-F4 All Subjects Notes
~ To be able to formulate objectives/hypothesis
~ To determine the respondents/resource persons.
~ To determine methods of data collection required.
~ To access the problem likely to be experienced in the area.
(ii) Give two methods you would use to collect information on pollution.
~ Interviewing
~ Taking photographs/video
recording
~ Measuring the extent of
polluted area
~ Administering
questionnaires
~ Tape recording.
(iii) State three follow up activities necessary for the study.
~ Analyzing data
~ Writing report
~ Giving relevant advice to the stake holders
~ Discussing the findings
~ Displaying photography, sketches from the study
area.
5. (a) Give four uses of vegetation you are likely to identify
during the study. (3mks)
~ Use as fodder.
~ Use for providing fruits/roots/vegetables as food.
~ Providing wood fuel.
~ Controlling soil erosion
~ Use of ornaments/beauty, aesthetics.
(b) Why is it necessary to sample part of the forest for the study?
~ In order to do a detailed study
~ To reduce cost of study

Page | 163
Prefer Calling Sir Obiero Amos @ 0706 851 439 for F1-F4 All Subjects Notes
~ To save time
~ The whole forest is too large to cover
within a day.
~ It would be less boring to study a small
area.
~ Some parts may inaccessible
~ To reduce bias
6. Types of field work
~ Field excursions
~ Field study
~ Field research
7. Importance of field work
~ Breaks classrooms monotony.
~ Make study of geography real.
~ Helps learner to acquire skills.
~ Encourages students to appreciate the
environment.
~ Enables learners to get first hand
information from the field.
~ Improves visual memory through
observation.
~ Enhances what has been learnt in class.

Page | 164
Prefer Calling Sir Obiero Amos @ 0706 851 439 for F1-F4 All Subjects Notes
CHAPTER 6
MAPS AND MAP WORK
Map of Kisumu East (1:50,000)
(a) (i) What is the bearing of the Trigonometrical station at grid reference 081980
from the rock outcrop at grid reference 071992. (2mks)
2. (a) (i) 139° 1° (138 - 140°)
(ii) Measure the length of the all weather road (bound surface) 1321, from, the
junction at grid reference 974911 to the edge of the map, grid reference 947967.
(2mks)
7.2 km 0.1 (7.1 - 7.3
km) (b) (i) Relief of the area
covered by the map.
~ The highest area is Nandi escarpment/187m above sea level.
~ The lowest area is to the south West which is about 1140 m above sea
level.
~ The east is a plain\Kanu plain/plateau
~ North western part is hilly with some steep slopes
~ To the North Eastern is the Nandi escarpment
~ The northern part is dissected by rivers
~ The South-west is a basin occupied by a lake.
~ There are numerous river valleys with steep sides in the highlands and
are broad in the lowlands.
(ii) Influence of relief on settlements.
~ The steep slopes/escarpment has been avoided because they are unsuitable
for construction of houses/for farming.
~ There are a few settlements on the hilly areas because the slopes are
gentler.
~ The plains are densely settled as the land is flat/gently sloping.
~ The basin are avoided as the land is water logged/flooded/swampy.
(c) Economic activity Evidence
- Quarrying - Quarry

Page | 165
Prefer Calling Sir Obiero Amos @ 0706 851 439 for F1-F4 All Subjects Notes
- Trading - Markets
- Transportation - Roads/railway/main tracks/foot
paths
- Processing - Sisal factory
- Manufacturing - Ginnery/flour mills
(d) (i) three findings they are likely to have come up with.
~ The river has many
meanders
~ The river has
tributaries/confluences
~ The river disappears into
a swamp
~ The river has a wide flood
plain
~ The river is at its old age
stage
(ii) Advantages of studying rivers through field work.
~ It enables students to relate what is learnt in classroom
to what is in the field.
~ Students are able to measure and calculate the velocity
of a river and its size.
~ Students are able to count the number of tributaries.
~ Students are able to gauge the impact of the river on
the area.
~ They are to find out the uses of the river.
~ It allows students to acquire appropriate attitude
towards environment.
~ It breaks classroom monotony for students and
teachers.

Page | 166
Prefer Calling Sir Obiero Amos @ 0706 851 439 for F1-F4 All Subjects Notes
~ It allows students to use their observation skills to
make conclusion.

CHAPTER 7
ROCKS AND MINERALS
1. (a) describe the following
characteristics of rocks; (i) Colour .
Distinct appearance by colour used to identify specific minerals
eg. Gold is yellow.
(ii) Cleavage.

Tendency of mineral to break in certain direction. Some minerals break


along planes on which atomic bonds are relatively weak.
(iii) Hardness.

Ability to resist scratching. Various minerals have varying degree of


hardness eg. Talc is softest while Diamond is hardest.
(b) (i) Give two types of igneous rocks
(2mks)
~ Volcanic rocks\extrusive igneous rock.
~ Plutonic rocks/intrusive igneous rocks.
(f) Uses of rocks
~ Some unique rocks e.g. crying stone of Kakamega present spectacular
scenery for tourist attraction which helps earn the country some foreign
exchange.
~ Rocks are parent material for soil formation exploited in agricultural
activities.
~ Valuable rocks and minerals such as gemstones and diamond are
exploited to generate income.
~ Rocks provide building and construction materials e.g. marble, ballast
and sand used in construction of houses.

Page | 167
Prefer Calling Sir Obiero Amos @ 0706 851 439 for F1-F4 All Subjects Notes
~ Rocks are useful as raw materials in construction industry e.g. the coral
rocks and coral limestone are used in manufacture of cement.
2. (a)State two characteristics of sedimentary rocks (2mks)
~ The rocks are formed from sediments of preexisting rocks.
~ Rock sediments are arranged in layers.
~ Processes involved act at ordinary temperatures
~ Sediments are non-crystalline
~ Some sediments contain fossils
~ Sediments are compressed, hardened and consolidated by cementing
material to form sedimentary
rock.
(b)Give two examples of chemically formed sedimentary rocks.
Trona, gypsum, flint, rock salt
3. (a) In each case name the type of rock which results from the metamorphism of:

(i) Granite→ Gneiss


(ii) Clay → slate
4. (a)(i) What is a rock? (2mks)
~ Rocks are naturally occurring agglomerations of mineral particles
forming part of the earth crust.
(ii) Describe three ways through which sedimentary rocks are formed
~ Mechanically formed sedimentary rocks formed from deposition of
sediments of other rocks in layers.
~ Organically formed - formed from remains of dead plants and animals
which are laid down in layers.
~ Chemically formed - formed from mineral particles dissolved from tend
and deposited in layers into water bodies.
(b)Describe two process through which sedimentary rocks changer into metamorphic
rocks
~ Weight of overlying layers cause change in grain arrangement in
dynamic metamorphism.

Page | 168
Prefer Calling Sir Obiero Amos @ 0706 851 439 for F1-F4 All Subjects Notes
~ Heat of magma get into contact with sedimentary rocks causing grains to
crystallize or form new minerals.
~ During mountain building rocks are compressed and heat generated in
thermodynamic metamorphism causing changes in structure and
recrystallization of minerals.
5. Differentiate between plutonic rocks and volcanic rocks.
~ Plutonic rocks are igneous rocks which form beneath earth surface when
magma cool slowly forming large crystals\course grained/course
textured.
~ Volcanic rocks are igneous rocks formed on the earth surface when lava
cool rapidly forming small crystals fine grained/textured.

Page | 169
Prefer Calling Sir Obiero Amos @ 0706 851 439 for F1-F4 All Subjects Notes
CHAPTER 8
MINING
1. Conditions necessary for the formation of petroleum.
~ Presence/deposition of remains of flora and fauna fossils over a
long period of time.
~ Presence of non porous rocks underneath the deposits of flora
and fauna
~ Deposition of other layers of rocks/ non -porous rocks over the
remains of flora and fauna.
~ Compression of remain of flora and fauna due to folding of the
layer of rocks.
2. (a) (i) Name the minerals mined in the areas marked S, T and V.
S - Oil/Petroleum

T - Bauxite/Gold

V – Diamond
(ii) State two formations in which mineral ores occur.
~ Some minerals occur as
evaporates.
~ Others occur as veins/lodes.
~ Some minerals occur as
alluvial deposits.
~ Some occur as weathered
products.
~ Some minerals are found in
seam
(b) Explain four problems which Zambia experiences in the exportation of copper.
~ Zambia is landlocked/ has no coastline hence copper has to pass
through other countries to reach the seaport.
~ The distance from Zambia to the coast is long which makes
transportation of copper expensive.

Page | 170
Prefer Calling Sir Obiero Amos @ 0706 851 439 for F1-F4 All Subjects Notes
~ Political instability in the neighbouring countries makes it insecure to
transport copper through them to the coast
~ Congestion at the seaports causes delays in loading and off-loading of
copper
~ Loss of copper through theft while on transit deprives Zambia of the
part of the expected revenue.
~ Copper is bulky thus it can only be transported by rail which is slow.
(c) Describe three negative effects of open cast mining on the Environment
~ The land is left with gaping quarries which are ugly interfere with the
natural beauty of the landscape.
~ The heaps of rock waste hinder any other forms of land use/create a
landscape that is expensive to rehabilitate/barren landscape.
~ The dust produced during the mining pollutes the atmosphere/is a
health hazard.
~ Open cast mining causes shortage of land as it hinders settlement/leads
to displacement/hinders agriculture.
~ Large scale blasting of rocks leads to instability of the basement rocks.
~ Water collects in the hollows left by open cast mines creating ponds
which become habitats for disease causing organisms
~ It interferes with the natural vegetation which is cleared before
extraction of the mineral begins/takes time to regenerate.

Page | 171
Prefer Calling Sir Obiero Amos @ 0706 851 439 for F1-F4 All Subjects Notes
Page | 172
Prefer Calling Sir Obiero Amos @ 0706 851 439 for F1-F4 All Subjects Notes

You might also like